Nothing Special   »   [go: up one dir, main page]

Grammarmerged

Download as pdf or txt
Download as pdf or txt
You are on page 1of 125

LESSON 1

Subject-Verb Agreement (Singular-Plural)

If I was only allowed to teach ONE grammar rule to any SAT or ACT prep student, this is the one
I would pick.

The name may be long, but it’s also perfect: “Subject-Verb Agreement (Singular-Plural)” - which
I will often abbreviate as “Subject-Verb Agreement” for the rest of this book. This rule is about
agreement between an essential pair of words that every sentence needs: a subject and a main
verb.

Every single sentence in English contains a subject and a main verb, so you can imagine why this
rule is so important.

What do the Subject and Main Verb have to “agree” on? That’s whether they are singular or
plural - “one” or “many”. The subject and main verb must both agree upon this.

A singular subject needs a singular verb. Plural subject, plural verb. It’s a very basic concept that
can be dressed up in all sorts of creative and challenging ways.

Let’s look at a simple sentence:

“Sam walks to the beach.”

The “Subject” is the main character; in this case, “Sam.” The main verb is “walks,” which is the
primary action that the subject is doing.

This sentence sounds fine, but what if we alter our sentence to:

“Sam walk to the beach.”

Sounds pretty terrible, doesn’t it?

Your ears know that a singular/plural error has happened. They just know!

In grammar-speak, here’s what went wrong. “Sam” is singular noun – one person. However,
“walk” is a plural verb, so they don’t match up. “Walks,” on the other hand, was singular, which
is why the original sentence sounded perfectly fine.

28
A Helpful Test
If you can’t tell the difference between singular and plural verbs, use “He blanks, they blank.”:

Test the sentence in your mind with both “he/she” and “they.” You’d say “he walks, they walk.”
Your inner ear will know which way is correct.

Is “swims” a singular or plural word? Try the test: “he swims, they swim.” Since “swims” sounds
right when matched with the singular “he,” we know that “swims” is a singular verb.

Is “jump” a singular or plural verb? “He jumps, they jump”: it’s plural, because it sounds right
when matched with the plural “they.”

Is “falls” singular or plural? How about “whistle”? (singular: “he falls,” then plural: “they
whistle.”)

I use this simple test throughout the SAT & ACT to help me figure out if a word is singular or
plural.

Telling the Difference Between Singular & Plural Nouns


You shouldn’t have much trouble determining if most nouns are singular or plural. However,
there are some “tricky” nouns that the SAT & ACT use to trap unwary students. Here are some
examples:

Tricky words that may sound plural, but are actually singular:

 Everyone  The group


 Team  The team
 Country  Pack (of dogs)
 The whole business  School (of fish)

All of these words are SINGULAR – just one team, just one school of fish, etc. – even though we
know that they are made up of many people or animals. The SAT will try to trick you like this:

“The entire team are packing their bags and going back home.”

In the above example, the question is trying to exploit confusion you may have about whether
“team” is singular or plural. Remember that we’re talking about just one team, even though it’s
made up of many players. Therefore the main verb should use the singular, and read:

“The entire team is packing its bags and going back home.”

29
Identifying “Compound” Plural Subjects
Sentences can include what I call “compound” plural subjects. This is when multiple singular
nouns combine to form one plural subject. For example:

“Sam and Bally walk to the beach.”

Now, even though both “Sam” and “Bally” are singular, they have been glued together with the
conjunction “and.” From now on, the sentence will be talking about both of these people
together, and two people make a plural subject. That’s why this sentence uses the plural verb
“walk,” not the singular “walks.”

If you didn’t pay attention to the “Sam and” at the beginning of the sentence, you would only
focus on “Bally walk to the beach.” That would sound wrong to you, and you’d probably change
“walk” to the singular verb form, like so:

“…Bally walks to the beach.”

Now this small section of the sentence sounds good, but if you plugged it back in and suddenly
noticed “Sam and” at the beginning of the sentence, you’d realize the Subject-Verb Agreement
was now wrong!

The SAT & ACT exploit this compound-subject trick by inserting giant piles of steaming
distraction in the middle of sentences (that’s why you apply Prelesson B on Eliminating Details!)

Here’s an example, based on our previous sentence. My alterations won’t change anything
about the basic structure of the sentence “Sam and Bally walk to the beach.” I’m just adding
bunch of junk details to distract you from the compound plural subject “Sam and Bally”:

“Sam, who had kept in touch with her old friend for some time (even after leaving
Florida), and, alongside her, her friend Bally, walk amidst the butterflies to the
isolated beach.”

WHEW. Now that sentence looks awful.

Doesn’t it sound wrong if you only focus narrowly on the portion “alongside her, her friend
Bally, walk amidst the butterflies”? Your ear will prefer the singular form “Bally walks amidst
the butterflies.” Resist that temptation, because really, there’s still a compound plural subject
and verb: “Sam and Bally (compound plural) walk.”

This is a sentence where Eliminating Detail will really bust through the fog. We can safely
ignore everything except a small portion of our original sentence: “Sam and Bally walk.”

30
Catching Compound Plural Subjects: Imagine the Action
Another tactic that helps me catch these mistakes is to actually visualize the sentence taking
place in my head. Is one thing doing the action (singular), or are two or more nouns involved
(plural)? Mentally picturing the sentence will often help answer this question.

A few more examples of compound plural subjects:

 “They and I”

 “The seagull, the raven, and the dove”

 “The cat, sitting in the meadow, and the dog, jumping at the gate” (There are
two animals. Get rid of details and you have “The cat and the dog.”)

 My brother, who is often found designing video games, my father, who loves
his classic cars, and my mother, who enjoys reading about philosophy” (There are
three people. Get rid of the details and you have “My brother, my father, and my
mother”)

Identifying “Trick” Compound Subjects


Occasionally, a sentence may appear to you at first to be a compound plural subject, but when
you eliminate unessential details, it turns out to be singular, not compound. Here’s an example:

The dog, along with his many canine friends, jumps in the water.

Looking at this sentence, it would seem that “many” dogs (plural) are jumping in the water.
That is, in fact, what is being described, but grammatically, the subject and verb of this
sentence are still singular.

That’s because “along with his many canine friends” is a nonessential modifying phrase (you’ll
learn to call this a “Parenthetical Clause” in Lesson 9 on Sentence Structure). It’s attached to
“the dog,” but this phrase is not essential to the structure of the sentence. You can eliminate
details and reduce the sentence to “The dog jumps in the water,” which is entirely singular.

It’s not part of a compound subject in the same way that “The dog and his many friends jump in
the water” would be. In this latter case, it would most definitely be a compound plural subject
and plural verb.

Continually use elimination of details to discover the essential structure of each sentence, then
properly match singular and plural to ensure that Subjects and Verbs can agree.

31
Subject-Verb Agreement Quick Reference
 Singular subjects go with singular verbs. Plural subjects go with plural verbs. This is the
among the most common topics on the SAT & ACT Grammar sections.

 Use the test “he blanks / they blank” (e.g. “he runs / they run”) and trust your ear to
determine if a verb is singular or plural.

 Eliminate details and use your logic to decide if the subject is singular or plural.

 Remember that some words might sound plural, but are really singular collective groups,
like “the entire company” or “the whole country.”

 Compound plural subjects are plural because they are made of multiple singular subjects.

 Sometimes, a nonessential descriptive clause can seem like part of a compound plural
subject at first - but nonessential clauses will never count towards the main subject, so
exclude them. Study Parenthetical Clauses in Lesson 9 on Sentence Structure.

OK, let’s look at some questions from the Pretest:

Our football team, currently in the running for the championship, and our
school tennis star, who has dominated her recent matches, is putting us in
the running for a record season this year.

(A) [NO CHANGE]


(B) matches, was putting us
(C) matches, are putting us
(D) matches, were putting us

OK – You should see the main verb “is” underlined and already be thinking, “what could go
wrong here?” Well, one common possibility is a Subject-Verb Disagreement error…

So, ask yourself “who or what is putting the school in the running for a record season”?
According to the sentence, that’s both the football team and the school tennis star.

32
Since we’re talking about both a team and a person, we have a compound plural subject, and
our main verb needs to be plural as well. However, in the sentence, the main verb “is” is
currently singular. That’s our classic Subject-Verb Agreement mistake: a plural subject (football
team and tennis star) mistakenly matched to a singular verb form (“is” instead of “are.”) The
subject and verb don’t agree on whether they should be singular or plural.

This eliminates both “singular verb” answer choices, which are Choice A (“is”) and Choice B
(“was”). Looking at our remaining options, they are both plural verbs (“are” and “were”) but
they have different Verb Tenses (see the next Lesson for more info). Looking at the timeframe
of the sentence, it’s more likely that we should stay with present tense (notice the clue words
“currently,” “recent,” and “this year”). Choice C is the correct answer, because it’s both plural
and present tense.

Here’s another example from the Pretest:

Joseph Wharton, along with his many successful business ventures, has
helped shape the history of American industry.

(A) [NO CHANGE]


(B) Wharton and his many successful business ventures has
(C) Wharton, including his many successful business ventures, have
(D) Wharton, along with his many successful business ventures, have

This is an example of a “trick” compound subject. It might look like “Joseph Wharton” and “his
successful business ventures” are the subject of the original sentence, but it’s just Joseph
Wharton that counts. As usual, stuff set between two commas is just unimportant detail - a
Parenthetical Clause (see Prelesson B on Removing Details and Lesson 9 on Sentence
Structure). Take this portion out of the sentence to simplify it, and you’ll be left with “Joseph
Wharton has helped shape…”

Your ears should tell you that’s correct, but you could also test like this: Is “Joseph Wharton”
singular or plural? One person, so singular. But is the verb “has” singular or plural? Test by
saying to yourself, “He has, he have.” Which one sounds correct? The first, singular version.

“Has” is singular, so it’s correctly matched with the singular subject “Joseph Wharton.” That
means Choice A, “No Change,” is quite possibly correct.

Choice B drops the commas around “business ventures,” making that middle section equal in
importance to the subject “Joseph Wharton” - this creates the compound plural subject
“Wharton and his ventures…” Now the subject really is plural, so the singular verb form “has”
suddenly creates a singular-plural disagreement.

33
Choice C uses the plural verb “have,” which disagrees with the singular subject “Joseph
Wharton.” It’s also an example of a Misplaced Modifier error (Lesson 11). The descriptive
phrase “including his business ventures” doesn’t make sense here, because this phrase is
modifying “Joseph Wharton,” but a person can’t contain or “include” business ventures. Read
Lesson 11 on Misplaced Modifiers for more info and practice.

Choice D again puts the middle section back into commas, creating a Parenthetical Clause as in
Choice A. This means the subject of the sentence remains Joseph Wharton (singular), but this
answer choices uses the plural verb form “have,” creating a Singular-Plural Disagreement error
again.

After evaluating our options, Choice A is correct and all the other answer choices are wrong!

34
Useful Links:
Redesigned SAT Practice Tests: http://www.cracksat.net/sat-tests/
SAT Online Practice Tests: http://www.cracksat.net/tests/
SAT Subjects Tests: http://www.cracksat.net/sat2/
SAT Downloads: http://www.cracksat.net/sat-downloads/
For more redesigned SAT information, please visit http://www.cracksat.net
SAT Grammar Practice Tests:

SAT Downloads:
Redesigned SAT Downloads: http://www.cracksat.net/sat-downloads/
SAT real tests download:
http://www.cracksat.net/sat-downloads/sat-real-tests.html
SAT official guide tests download:
http://www.cracksat.net/sat-downloads/sat-official-guide-tests.html
SAT online course tests download:
http://www.cracksat.net/sat-downloads/sat-online-course-tests.html
SAT subject tests download:
http://www.cracksat.net/sat-downloads/sat-subject-tests.html
PSAT real tests download:
http://www.cracksat.net/psat/download/

1000+ College Admission Essay Samples:


http://www.cracksat.net/college-admission/essays/
LESSON 2
Verb Tense & Form

The next grammar concept on the SAT & ACT is relatively simple for the majority of students.
This is the concept of Verb Tense and Form.

First of all, I recommend reviewing Prelesson A on Parts of Speech, particularly the information
on verbs. Remember that verbs are the “doing” or “being” words in English. They show actions
(like “the dog barks) or “states of being” (like “the cat is happy”).

Verbs are also the words in English responsible for showing when the action of the sentence is
happening. This concept is called “Verb Tense.” There are “future tense,” “present tense,” and
“past tense” versions of every verb, and other more-advanced tense options as well.

Here are some simple examples of the most common verb tenses:

 I run (present tense)

 I ran (past tense)

 I will run (future tense)

The main source of Verb Tense questions on the SAT & ACT tests arise from an incorrect
sequence of events, for example:

“Once I have my own car, I had driven to the movie theater.”

In this example, the past-tense verb “had driven” is directly contradicting the timeline of the
sentence - a timeline that shows the narrator does not have a car yet. So, it would be logically
impossible to already have driven to the movie theater in the past. This is an example of a basic
Verb Tense error: the timeline doesn’t make sense, because a verb is in the wrong tense.

Let’s fix the situation by changing the verb to future tense.

“Once I have my own car, I will drive to the movie theater.”

44
Most Verb Tense errors are easy to find and explain - just stay on the lookout for any illogical
timelines or sequences.

Verb Form is similar to Verb Tense, but slightly different. In fact, sometimes I just use “Verb
Form” as my general name for other verb-based mistakes. Here’s an example of what I call a
“Verb Form” mistake:

“Please let me to go shopping!”

In this case, we should not use the infinitive verb form “to go”, but rather the simple present
tense form “go.”

“Please let me go shopping!”

Note: Making the correct choice of Verb Tense & Form is often linked to the rule of Parallelism
(see Lesson 10). Many questions that initially look like Verb Tense can turn out to actually be
about Parallelism. Look for both of these rules to appear together often!

Now let’s look at a simple example from the Pretest:

Project SCORE was the first communications satellite ever put into Earth’s
orbit, and it will be launched on December 18, 1958.

(A) [NO CHANGE]


(B) it was launched
(C) they were launched
(D) it is launched

Notice that the main underlined portion of this sentence focuses on a verb, “will be launched,”
and the four answer choices mainly make changes to the verb tense. We are given options for
past tense (“was launched” and “were launched”), present tense (“is launched”), and future
tense (“will be launched”).

The timeline of the sentence makes clear that this satellite launch happened in the past (in
“1958”), so we can remove all present (Choice D) and future (Choice A) options.

Between the remaining options, Choice B and C are both past tense (which we want), but
Choice C commits a Pronoun-Antecedent Agreement error (read Lesson 5 for more
information: the antecedent “satellite” must be replaced by the singular pronoun “it,” not the
plural pronoun “they”).

That leaves only Choice B as the correct answer, because it matches the correct Verb Tense
(the past tense) with a singular pronoun.

45
Verb Tense & Form Quick Reference

 Verb Tense primarily has to do with the order of events in a sentence or paragraph’s
timeline.

 When determining the timeline, context matters - find clues to the time periods and order
of events within the sentence or paragraph.

 Parallelism (see Lesson 10) often has an influence on the correct choice of Verb Tense &
Form. Be sure to study Parallelism carefully, and keep an eye out for it whenever you think
you’re looking at a Verb Tense and Form question.

Now let’s look at one more example from the Pretest:

Plymouth was once a brand of automobiles produced in the United States,


but it was dissolved and absorbed into other brands in 2001; however,
some investors would like to revive the marque, leaving the future of the
Plymouth name in question.

(A) [NO CHANGE]


(B) will revive
(C) revived
(D) reviving

Notice that the underlined portion and our possible answer choices are playing with the tenses
and forms of the verb “revive.” Take context clues from the rest of the sentence: the brand was
dissolved in 2001 (in the past), but some investors might revive the brand (a future possibility).
That eliminates Choice C, which is a past tense verb.

Choice D causes a Sentence Structure mistake (see Lesson 9) by removing the main verb
“would like” and not replacing with another main verb, thus destroying the Independent Clause
that must follow the semicolon (also see Lesson 8 on Punctuation Marks).

Between Choice A and B, these are both future tense forms. However, Choice B implies that
this brand’s revival is a sure thing (“will revive”), which conflicts with the meaning of the “future
being in question” - while Choice A correctly leaves open the possibility of revival, without
guaranteeing it (“would like to revive”). Choice A nails the correct verb tense and form, which
makes it the right answer.

46
Useful Links:
Redesigned SAT Practice Tests: http://www.cracksat.net/sat-tests/
SAT Online Practice Tests: http://www.cracksat.net/tests/
SAT Subjects Tests: http://www.cracksat.net/sat2/
SAT Downloads: http://www.cracksat.net/sat-downloads/
For more redesigned SAT information, please visit http://www.cracksat.net
SAT Grammar Practice Tests:

SAT Downloads:
Redesigned SAT Downloads: http://www.cracksat.net/sat-downloads/
SAT real tests download:
http://www.cracksat.net/sat-downloads/sat-real-tests.html
SAT official guide tests download:
http://www.cracksat.net/sat-downloads/sat-official-guide-tests.html
SAT online course tests download:
http://www.cracksat.net/sat-downloads/sat-online-course-tests.html
SAT subject tests download:
http://www.cracksat.net/sat-downloads/sat-subject-tests.html
PSAT real tests download:
http://www.cracksat.net/psat/download/

1000+ College Admission Essay Samples:


http://www.cracksat.net/college-admission/essays/
LESSON 3
Adjectives & Adverbs

As we know from Prelesson A on Parts of Speech, Adjectives are words that are used to modify
nouns - to describe or add detail about nouns or pronouns. They tell us more about what
something or someone is like. A few examples of Adjectives are “blue,” “fast,” “small,”
“important,” and “complete.”

We also learned that Adverbs are words that modify verbs, adjectives, or other adverbs.
Adverbs tell us more about how something is done or further clarify other modifiers. A few
examples are “quickly,” “fundamentally,” “importantly,” “completely,” “extremely,” and “very.”

You can imagine adjectives and adverbs as twins separated at birth and raised in different ways.
They have something in common: they both modify - describe and add details - to other words.

However, just like twins raised separately could have very different favorite foods, adjectives
will only play nicely with one other type of words: Nouns. On the other hand, Adverbs are more
omnivorous. They will play nicely with verbs, adjectives, and adverbs -but never with Nouns!

This may seem like a ridiculous distinction, but unfortunately, it’s the way the grammar game is
played in English. Therefore, for the SAT & ACT tests you need to be able to identify adjectives
vs. adverbs and determine if they are “attached” to the right type of word – that is, are they
modifying the correct types of words that each of these “twins” plays nicely with?

Let’s take a closer look at the difference between Adjectives and Adverbs…

51
Telling the Difference Between Adjectives & Adverbs
The easiest common test for whether a modifying word is an adverb or an adjective is to look
for an “-ly” at the end of the word. 99 times out of 100, an adverb will look like an adjective
with “-ly” stuck on at the end of it.

Here are a few examples of regular, everyday Adjective / Adverb pairs:

The adjective “wise” becomes the adverb “wisely.”

The adjective “sad” becomes the adverb “sadly.”

The adjective “haughty” becomes the adverb “haughtily.”

A very few adverbs are irregular: for example, the adjective “good” becomes the irregular
adverb “well.”

Here’s the good news: the SAT & ACT virtually never test irregular adverbs!

Within the Adjectives & Adverbs topic, your job on the SAT or ACT is to identify these modifying
words when they are underlined and make sure that adjectives and adverbs are each only
modifying the types of words they’re meant to play nicely with.

Adjectives & Adverbs Quick Reference


 Adjectives and adverbs both modify - add description and detail to other words - but
they only “play nice” with their own favorite types of words.

 Adjectives only play with Nouns. Adverbs can play with Verbs, Adjectives, and other
Adverbs but not Nouns. If you’re confused, review Prelesson A on Parts of Speech!

 Any adverbs tested on the SAT & ACT are always regular and can be identified by the
“-ly” at the end of the word.

 The SAT & ACT do not (or very, very rarely) test irregular adverbs.

 Most regular adverbs can be formed just by adding “-ly” to the end of an adjective.

 Be suspicious of descriptive or modifying words and double-check your common


sense to identify what a given adjective or adverb is describing.

52
Let’s look at a couple of examples from the Pretest:

Christina Wahlberg can handle a foil quite deft; in fact, she has competed
in Olympic fencing.

(A) [NO CHANGE]


(B) can deft handle a foil
(C) can handle a foil quite deftly
(D) being a deft foil-handler

In the underlined portion of this sentence, we see the adjective “deft” (meaning “skillful”).
Should we leave it as-is, or must it be changed to its adverb form, “deftly”? To answer that
question, we must identify the word within the sentence that “deft” is meant to modify.

First, apply some common sense – a “foil” is the type of sword used in fencing. Can an object,
like a sword, be “skillful”? Nope! Swords just lie on the table until someone picks them up – it’s
the person using the sword who is skillful or “deft,” not the sword itself.

However, in our sentence, “deft” is trying to modify how Christina can handle a foil. “Deft” isn’t
modifying a noun – the sentence does not say that “Christina is deft,” but that she “can handle
a foil” with skill. It’s the action of “handling” that’s being described. We’re modifying a verb, not
a noun. That means we need an adverb, not an adjective.

Therefore, Choice A is out, because it uses an adjective (“deft”) to modify a verb (“can handle”).
Choice B makes the same mistake, just with a different word order.

Choice D switches up the wording, and properly uses the adjective “deft” to modify the noun
“foil-handler,” but also results in Wordiness (see Lesson 13) and an incomplete sentence before
the semicolon (see Lesson 9 on Sentence Structure and Lesson 10 on Punctuation Marks).

Choice C changes very little from the original sentence – it merely adds “-ly” to “deft” in order
to transform it into the adverb “deftly” – but fortunately, that’s all we need to fix this problem.
Now the adverb “deftly” is being properly used to modify the action verb “can handle.” Choice
C is correct!

53
Here’s another example from the Pretest:

An ocean mammal, such as the blue whale, behaves quite differently than
mammals that live on land.

(A) [NO CHANGE]


(B) behave quite differently
(C) behaves quite different
(D) has behaviors quite differently

We can probably tell this is an Adjectives & Adverbs question because of the little “-ly” that is
playing such a big role in the different answer choices.

First, look at Choice D. It’s one of our worst options: the adverb “differently” is attempting to
modify the noun “behaviors,” and by now we know that adverbs cannot be used to modify
nouns - only adjectives can modify nouns.

Now, take a look at Choice C. “Different” is an adjective, but the word is modifying how the
whale “behaves,” which is a verb! No way can we use an adjective to describe a verb – we
would need an adverb instead.

We could repair this by changing “different” to its adverb form “differently.” That leaves us
with Choices A and B. One problem - Choice B commits a Subject-Verb Agreement error (see
Lesson 1) by pairing a plural verb (“behave”) with a singular subject (“an ocean mammal.)

That leaves Choice A, “No Change,” as the correct answer. It uses an adverb, “differently” to
modify a verb, “behaves,” and also avoids any singular-plural mistakes.

54
Useful Links:
Redesigned SAT Practice Tests: http://www.cracksat.net/sat-tests/
SAT Online Practice Tests: http://www.cracksat.net/tests/
SAT Subjects Tests: http://www.cracksat.net/sat2/
SAT Downloads: http://www.cracksat.net/sat-downloads/
For more redesigned SAT information, please visit http://www.cracksat.net
SAT Grammar Practice Tests:

SAT Downloads:
Redesigned SAT Downloads: http://www.cracksat.net/sat-downloads/
SAT real tests download:
http://www.cracksat.net/sat-downloads/sat-real-tests.html
SAT official guide tests download:
http://www.cracksat.net/sat-downloads/sat-official-guide-tests.html
SAT online course tests download:
http://www.cracksat.net/sat-downloads/sat-online-course-tests.html
SAT subject tests download:
http://www.cracksat.net/sat-downloads/sat-subject-tests.html
PSAT real tests download:
http://www.cracksat.net/psat/download/

1000+ College Admission Essay Samples:


http://www.cracksat.net/college-admission/essays/
LESSON 4
Pronoun-Antecedent Clarity

This topic of this lesson is called “Pronoun-Antecedent Clarity,” and it’s the first of three major
pronoun-based rules on the tests. Pronoun mastery will be responsible for quite a lot of points
on your SAT or ACT Grammar score!

First, let’s quickly review what we know about pronouns from Prelesson A: Parts of Speech.

Pronouns are the short, “anonymous” words like it, he, them, and I. These words act just like
Nouns in many ways. However, they also must follow some important special rules.

Pronouns are useful words for shortening our sentences, avoiding repetition, and not having to
say the same names, objects, and places over and over again. Indeed, that is their most
valuable asset - pronouns save us time and repetition.

On the other hand, the downside of using pronouns is that they can be vague and confusing.

That’s why I sometimes call pronouns “anonymous nouns” - they replace specific nouns with
more general and vague words like “they” or “it.”

For example: “My dog is running around. She is very excited.” The specific noun “dog” is
replaced by the generic pronoun “she.”

Remember: There is a special term - the antecedent - for any noun that is replaced by a
pronoun. In the example in the line above, we would refer to the noun “dog” as the antecedent
of the pronoun “she.”

WARNING: Pay special attention to the note in bold about the antecedent. Make sure you
understand exactly what an “antecedent” is before continuing!

Now, once you understand:


A) What a pronoun is…
B) What an antecedent is, and…
C) Why using pronouns without clear antecedents is confusing…

Then this rule is easy, because Pronoun-Antecedent Clarity is nothing more than the
requirement to have a clear and definite antecedent to match with each and every pronoun we
encounter on the SAT & ACT grammar tests.

60
For example, if we were to write:

Between cats and dogs, James likes them better.

Then we have a problem: the pronoun “them” does not have a clear antecedent. Are we
referring to “cats” or to “dogs” as the animal that James prefers?

This is just a simple example of the Pronoun-Antecedent Clarity rule being broken. By adding
details and nonessential parenthetical clauses (see Prelesson B on Eliminating Details and
Chapter 9 on Sentence Structure) to distract your attention, the SAT & ACT tests can make this
simple pronoun mistake much harder to find.

Be sure you are looking for these Pronoun Clarity issues in advance. You already know they will
be on the test, so predict and anticipate them. It’s much easier to notice Pronoun-Antecedent
mistakes if you’re already looking out for them at all times.

Let’s take a look at an example of Pronoun-Antecedent Clarity from the Pretest:

Cyclone Althea devastated portions of northern Australia in December 1971;


the strong winds ripped roofs off houses, hail damaged vehicles and windows,
and it cost the Australian federal government close to a billion dollars in
modern-day adjusted values.

(A) [NO CHANGE]


(B) and they cost
(C) and those cost
(D) and the storm cost

Notice the pronoun “it” in the underlined portion, and other pronouns throughout the answer
choices: “they” and “those.” Choice D, on the other hand, replaces the pronoun with the
specific noun “the storm.” Could there be a reason?

There certainly is. The three pronoun choices all share the same problem: none of them have a
clear and definite antecedent. Does “it” refer to the cyclone, or to the winds and hail? What
about “they”, or “those”? All three pronoun-based choices have the same issue: the antecedent
to the pronoun is not clear enough.

Choice D is the correct answer - it sidesteps the problem entirely by not using any pronoun at
all, and replaces instead with a specific noun, “the storm.”

61
Pronoun-Antecedent Clarity Quick Reference
 A pronoun is an “anonymous noun” (like “it”) that replaces a specific noun (like “table.”)

 The antecedent is the specific noun (like “table”) that is replaced by a pronoun (like “it.”)

 Every pronoun must have a specific and clear antecedent.

 If the antecedent of a pronoun is ever unclear, this rule is being broken.

 Get these questions right by anticipating and looking for them in advance. Any time a
pronoun is part of a grammar question, double-check that it actually has a clear antecedent.

Now let’s try another Pronoun-Antecedent Clarity question from the Pretest:

Historical wolf hunts would include noblemen, conscripted peasant


farmers, and trained wolfhounds, though they were often injured as the
wolves attempted to defend themselves with teeth and claws.

(A) [NO CHANGE]


(B) it was
(C) the wolfhounds were
(D) these were

Explanation: Remember to anticipate Pronoun Clarity errors any time the sentence uses a
pronoun, especially in an underlined portion. In this case, we see the pronoun “they” is
underlined, along with replacement choices “it” and “these.” As in our previous Pretest
example, none of these pronouns are sufficiently clear. Do they refer to the entire hunting
group: noblemen, farmers, and wolfhounds? Or is there a more specific antecedent who is
being injured by the wolves?

Luckily, Choice C gives us a great option for eliminating any Pronoun Clarity issues by replacing
the pronoun with a specific noun, “the wolfhounds.”

It’s not uncommon for the SAT and ACT Grammar tests to resolve Pronoun Clarity mistakes by
simply removing the pronouns entirely and replacing with specific nouns. In this fashion, we can
completely remove any possibility of confusion arising from pronoun usage.

62
Review & Encouragement
It’s wise to always remain hyper-vigilant towards pronouns. Remember, Pronoun-Antecedent
clarity is only the first of three major rules for pronouns.

You can never go wrong by noticing and evaluating the use of pronouns on a grammar test.
Remain on your guard whenever pronouns are part of a question!

63
Useful Links:
Redesigned SAT Practice Tests: http://www.cracksat.net/sat-tests/
SAT Online Practice Tests: http://www.cracksat.net/tests/
SAT Subjects Tests: http://www.cracksat.net/sat2/
SAT Downloads: http://www.cracksat.net/sat-downloads/
For more redesigned SAT information, please visit http://www.cracksat.net
SAT Grammar Practice Tests:

SAT Downloads:
Redesigned SAT Downloads: http://www.cracksat.net/sat-downloads/
SAT real tests download:
http://www.cracksat.net/sat-downloads/sat-real-tests.html
SAT official guide tests download:
http://www.cracksat.net/sat-downloads/sat-official-guide-tests.html
SAT online course tests download:
http://www.cracksat.net/sat-downloads/sat-online-course-tests.html
SAT subject tests download:
http://www.cracksat.net/sat-downloads/sat-subject-tests.html
PSAT real tests download:
http://www.cracksat.net/psat/download/

1000+ College Admission Essay Samples:


http://www.cracksat.net/college-admission/essays/
LESSON 5
Pronoun-Antecedent Agreement (Singular-Plural)

This next topic - called Pronoun-Antecedent Agreement (Singular-Plural) - is closely related to


the lessons on Subject-Verb Agreement (Lesson 1) and Pronoun-Antecedent Clarity (Lesson 4).
Like those previous topics, Pronoun-Antecedent Agreement (Singular-Plural) deals with proper
selection of singular and plural forms of nouns & pronouns.

Now that we’re on our second pronoun-based lesson, let’s get one thing out of the way:

Pronouns on the SAT & ACT Grammar sections are ALWAYS SUSPICIOUS. They are
GUILTY until proven INNOCENT.

That’s because there’s just so darn much that can go wrong when pronouns are involved.(Now
might be a good time to review pronouns in Prelesson A, Parts of Speech, if you have any
questions about what “pronouns” or “antecedents” are).

The first and foremost rule of pronouns - as we just learned in Lesson 4 on Pronoun-
Antecedent Clarity - is: it must always be clear what antecedent noun a pronoun is referring to.
Because pronouns like it and she are “anonymous” by nature, there is plenty of room for
confusion.

If you can’t tell exactly what antecedent noun a pronoun is referring to, then it’s already wrong
because of Pronoun-Antecedent Clarity. Here’s an example of such a situation to review:

“Mr. Johnson and Mr. Daniels both agree that he should go to the party after all.”

“He” could refer to either Mr. Johnson or Mr. Daniels, so this sentence contains a pronoun
clarity error!

But, that’s just a review of our previous lesson on Pronoun-Antecedent Clarity. So it is to the
next topic that we turn our attention: Pronoun-Antecedent Agreement.

The name of this grammar rule actually explains itself perfectly:

Pronouns must agree with the antecedent noun they replace in singular/plural!

Some pronouns are plural, like “they.” Some pronouns are singular, like “it.”

Many times you can use common sense. “He” can only refer to one man at a time, so he is a
singular pronoun. “They” replaces several people, so they is a plural pronoun. “It” is a singular
pronoun. “We” and “us” are plural because they involve two or more people at a time.

70
Let’s look at an easy example of a Pronoun-Antecedent Agreement mistake:

“Daniel and Jeremy both went to the ice cream shop for treats, where he got one
three-scoop cone and one ice-cream sundae.”

Step 1: Be suspicious of all pronouns. I see the pronoun “he.” This could cause trouble.

Step 2: Identify the antecedent noun(s). In this case, the logic of the sentence shows that two
guys went to the ice cream shop, where they each got an ice cream treat. The singular pronoun
“he” is referring back to the plural antecedent “Daniel and Jeremy.”

Step 3: Check singular/plural agreement. “He” is a singular pronoun, but “Daniel and Jeremy” is
a compound plural antecedent. Big mistake! We must change “he” to “they” to match a plural
pronoun with a plural antecedent.

Be careful of certain singular nouns that may sound plural to you. This overlaps nicely with
Subject-Verb Agreement (Lesson 1), and it never hurts to review that chapter.

Basically, to some students, some nouns like “company,” “country,” “school (of fish),” and
certain other words sound plural because a company is made of many employees, a country has
many people in it, and a school is a group of many fish.

However, these nouns are, in fact, singular – ONE company, ONE country, ONE school of fish. If
the words were meant to be plural, they would take the plural form: companies, countries,
schools of fish. Just be aware that the SAT & ACT will often try to exploit this trick on the
grammar and English sections.

All Pronouns acting as subjects must agree with their verbs in Singular/Plural. This is exactly the
same rule as Subject-Verb Agreement (Lesson 1). Since pronouns can be subjects, singular
pronouns that act as subjects must connect to singular verbs, and plural pronouns must match
with plural verbs.

“Tricky” Pronouns: Singular or Plural?


The SAT & ACT don’t just test the easy & obvious pronouns, like “he” and “us.” There are also
trickier pronouns. Some of these are obviously singular or plural, but others are a little tougher
to figure out. Here are some examples:

Easier: One (singular), those (plural), all (plural), many (plural), few (plural)

Harder: No one, neither, either, anyone, everyone, someone, every, each

71
The bottom list is slightly tougher, but here’s something interesting: all of those “harder”
pronouns are singular. Many of them have a clue built in: “no one,” “anyone,” “everyone,” and
“someone” all include “one” in them. That’s a dead giveaway that they are one - singular!

“Every” and “each” also remind me of the word “one” (every one, each one).

“Neither” and “Either”: Singular or Plural?


The only pronouns that I think are really tricky are NEITHER and EITHER.

“Neither” and “neither/nor” can be singular or plural depending on the situation. They will
actually follow the second noun when determining their singular/plural nature.

For example, “Neither the cat nor the dogs” is plural. “Neither the dogs nor the cat” is singular.

Luckily, “either” follows the exact same rules as “neither.”

Now let’s look at Pronoun-Antecedent Agreement question from the Pretest:

Despite the group’s many musical successes, they are struggling to afford
new instruments and gas money for the van.

(A) [NO CHANGE]


(B) they struggle
(C) it is struggling
(D) he and she are struggling

Step 1: Be suspicious of pronouns. I see the pronoun “they” hanging out in the underlined
portion, so it might be causing trouble.

Step 2: Make sure you know exactly what antecedents the pronouns refer back to. I feel
confident from the context that “they” is clearly meant to refer to “the group.” No problem
here. (Notice that this also takes care of Lesson 4, Pronoun-Antecedent Clarity.)

Step 3: Check singular/plural agreement. The pronoun “they” is plural, but it’s being used to
replace a singular antecedent noun, “the group.” Instead of the plural form “they are,” we need
a singular form like “it is.” The correct answer is Choice C.

Note that Choice D, “he or she,” is also considered a singular pronoun, but “he or she” is not an
appropriate replacement for the word “group.”

72
Pronoun-Antecedent Agreement (Singular-Plural)
Quick Reference
 Pronoun-Antecedent Agreement is closely related to Subject-Verb Agreement (Lesson 1)
because they are both about “number” (i.e. “singular-plural”). These two topics together
make up a significant portion of the SAT & ACT English grammar tests.

 As we’ve already learned, pronouns’ jobs are to replace specific antecedent nouns.

 Because of their multiple risk factors, you must treat pronouns on the SAT & ACT as guilty
until proven innocent.

 If it is unclear exactly what antecedent a pronoun is replacing in the sentence, it’s already
wrong (Lesson 4 on Pronoun-Antecedent Clarity).

 Pronouns must match the antecedent noun they replace in singular/plural.

 Pronouns acting as subjects must also match their verbs in singular/plural.

 Many pronouns are obviously singular/plural, but some are trickier. Review them. The
presence of “one” (anyone, no one, someone) is an indicator that the pronoun is singular.

 “Neither” and “either” are special pronouns - they follow the second noun’s singular/plural
number.

73
Here’s another example from the Pretest:

Neither the duck nor the goose is able to enjoy long periods of time without
a nearby body of water.

(A) [NO CHANGE]


(B) nor the goose
(C) or the goose is able to
(D) nor the goose are able to

Remember what I said about the word “neither” being a tricky pronoun? When using “either”
or “neither,” the pronoun takes its singular or plural from the second noun in the list. (In this
case, “the goose” is the second noun.)

So, if “neither” is a singular pronoun in this case because “goose” is singular, then we need a
singular verb to match with it. (If this is confusing, review Lesson 1 on Subject-Verb Agreement).

Two of our choices use plural verbs: Choice D (“are able”) and Choice B (“enjoy”). So, those are
eliminated.

Only Choices A and C use the singular verb “is able.” However, Choice C uses the wrong pair of
conjunctions, “neither - or,” when the pair must be “neither - nor.”

Choice A is our only remaining option - it’s singular and uses the proper conjunctions - so the
right answer must be No Change.

74
Useful Links:
Redesigned SAT Practice Tests: http://www.cracksat.net/sat-tests/
SAT Online Practice Tests: http://www.cracksat.net/tests/
SAT Subjects Tests: http://www.cracksat.net/sat2/
SAT Downloads: http://www.cracksat.net/sat-downloads/
For more redesigned SAT information, please visit http://www.cracksat.net
SAT Grammar Practice Tests:

SAT Downloads:
Redesigned SAT Downloads: http://www.cracksat.net/sat-downloads/
SAT real tests download:
http://www.cracksat.net/sat-downloads/sat-real-tests.html
SAT official guide tests download:
http://www.cracksat.net/sat-downloads/sat-official-guide-tests.html
SAT online course tests download:
http://www.cracksat.net/sat-downloads/sat-online-course-tests.html
SAT subject tests download:
http://www.cracksat.net/sat-downloads/sat-subject-tests.html
PSAT real tests download:
http://www.cracksat.net/psat/download/

1000+ College Admission Essay Samples:


http://www.cracksat.net/college-admission/essays/
LESSON 6
Pronoun Case

“Pronoun Case” is the topic that covers the common question of whether we should say “my
brother and I go to the park” or “My brother and me go to the park” - and many other
situations as well.

There are two ways to deal with this common error on the SAT & ACT, and in real life as well:

Method #1: Test each noun by itself and trust your ear (about 90% effective) or
Method #2: Think about it the “grammar teacher” way (this is harder to learn, but more
powerful – and allows you to achieve 100% accuracy and certainty)

I’ll explain both ways, and you can pick the one you prefer. I myself use a blend of both – I trust
my trained ear nine times out of ten, but when I’m uncertain, I take a “grammar teacher”
approach to support my decision.

Approach #1: Try Each Noun By Itself & Trust Your Ear
This first style of solving Pronoun Case issues focuses on four steps:

1. Noticing the presence of pronouns in the sentence,


2. Trimming the sentence down to its basics,
3. Occasionally rearranging word order without changing sentence meaning, and
4. Trusting your “inner ear” to determine the right pronoun to use (he/him? They/them? etc.)

We will practice each of the four steps when we look at the questions from the Pretest, but for
now let me lay them out for you:

Step 1: You should always be looking for pronouns on the SAT & ACT English grammar tests!
This step overlaps with at least three other important topics: Subject-Verb Agreement (Lesson
1), Pronoun Clarity (Lesson 4) and Pronoun Agreement (Lesson 5). Every pronoun should be
closely examined!

Step 2: Practice eliminating unnecessary detail (Prelesson B). As always, this is an important
tool you can use on every single SAT & ACT grammar question, so you should always eliminate
details, even if you are “pretty sure” you know what the grammar mistake is. You may be
surprised once the details are removed and the core of the sentence stands exposed.

83
Step 3: When the SAT & ACT writers test Pronoun Case, they sometimes make the sentence
appear more complicated than it really is by choosing a word order that is unnatural, compared
to your everyday speech. In such cases, you can beat them at their own game by applying
Prelesson C and re-rearranging the order of the sentence into a more familiar-sounding style of
speech.

Step 4: “Trust your inner ear.” Try changing the pronoun’s case (more details on this in the next
half of this chapter) in the reduced, re-ordered sentence. Then use the case that “sounds
better” to you. You won’t get it right every single time with this method, but you’ll get it right a
lot of the time.

Pronoun Case Quick Reference


 Always remain suspicious of pronouns!

 Pronoun Case on the SAT & ACT is often about choosing between “I” and “me” (or between
“us” and “we.”)

 Method 1: Eliminate unnecessary details and rearrange the sentence (apply Prelessons B and C);
then, trust your inner ear. This will work about 90% of the time.

 Method 2: If you want to get this topic right 100% of the time, you’ll need to study up on the
difference between subjects and objects and the pronoun subject/object chart (coming up next
in this chapter).

84
Now we’re going to practice the first Pronoun Case method on a Pretest question:

Charles and me love to play dodgeball, and we have spent many an hour
engaged in this exciting pastime.

(A) [NO CHANGE]


(B) Charles and I love
(C) Charles and me loves
(D) Charles and I loves

Step 1: Notice the pronouns: “me” and “we.”

Step 2: Eliminate Details, with a twist.

First of all, we can reduce this sentence to “Charles and me love to play, and we have engaged.”

Now the twist: Try both “Charles” and “me” by themselves. You will have to change the plural
verb “love” to the singular form “loves,” but that’s fine - it’s just a natural result of applying this
pronoun test.

So, you’d try saying these two ways to yourself:

“Charles loves to play”/ “Me loves to play.”

Doesn’t that second version sound awful? Yes, because it’s wrong, and your ear knows it!

Step 3: Rearrange the word order if necessary. (No need to do that here; the sentence structure
is already in the most basic order.)

Step 4: Try saying the “problem pronoun” both ways. So you’d try:

“Me loves to play” / “I love to play.”

First one sounds awful, second one sounds good – so trust your ear; we should use the “I” form
of the pronoun, not the “me” form. That eliminates Choice A and Choice C.

We still have Choices B and D, but Choice D contains a Subject-Verb Agreement error (Lesson 1)
between a plural subject and a singular verb: “Charles and I loves.” (You should be getting good
at this Subject-Verb Agreement stuff by now!)

That leaves Choice B as the correct answer, because it uses the correct Pronoun Case and
avoids any singular-plural mistakes.

85
Love the SAT Test Prep

Simplifying and Reorganizing Sentences


Let me show you some quick simplifying and reorganizing tactics on this next Pretest sentence:

“Is it right that us workers must suffer due to incompetent management?”

Eliminating details gives:

“Is it right that us must suffer?”

Rearranging and simplifying again gives the core of the sentence:

“Must us suffer?”

This sounds awful. Try it the other way:

“Must we suffer?”

Now it sounds much improved, and it is in fact correct this way.

Approach #2: The Grammar Teacher Way


Now, if you want to catch Pronoun Case errors every single time, you need to understand this
rule the “grammar teacher way.” What you need to learn (or review) first is the concept of
“Object” vs. “Subject.”

Every complete sentence has a subject. The subject does the action - for example:

“The dog barks.”

More complex sentences also have objects, for example:

“The dog barks at the mailman.”

Objects are required for situations where someone does something to something else.

The dog barked “at” the mailman; barking was the thing done to the mailman, so he’s the
“object.”

The “subject” is the dog, the main character, who happens to be doing the action like a subject
should.

86
Love the SAT Test Prep

You usually don’t have to think about this stuff, except when you’re using pronouns like “it”
instead of dog and “him” instead of mailman. Let’s see what that would look like:

“It barks at him.”

OK, on this SAT or ACT this sentence would be wrong already because of Pronoun-Antecedent
Clarity issues (Lesson 4). However, let’s set that issue aside for a moment and deal only with
Pronoun Case. Here’s where we pull out our handy mental chart of pronoun case…

Oh wait, you don’t have one? OK, you can borrow mine for the moment! Make sure to get your
own (i.e. memorize this chart if you’re shaky on it):

Subject Pronoun Object Pronoun

I Me

He, She, It Him, Her, It

We Us

They Them

This chart represents a written version of “trusting your ear” from the previous four-step
approach to Pronoun Case. A lot of the time, your brain will just “know” this stuff through years
of practice and experience with the English language.

Now, can you add up the pieces of the “grammar teacher” way to handle Pronoun Case
questions?

1) We know how to tell the difference between a subject and an object.


2) We have a chart that tells us which pronoun to use depending on if it’s a subject or
object.

The dog was the subject, and has been replaced correctly with a subject pronoun from our chart:
“It.”

The mailman was the object, and has been replaced correctly with an object pronoun from our
chart: “Him.”

Therefore, the sentence “It barks at him” is correct in regards to pronoun case.

But, if we changed the sentence to “It barks at he,” for example, we would have a Pronoun Case
problem, because we replaced the mailman (who is an object in the sentence) with a subject
pronoun (“he”). Notice how the chart also supports what I’m saying.

87
Love the SAT Test Prep

Let’s try the “grammar teacher” method on the following Pretest question:

Is it right that us workers must suffer due to incompetent management?

(A) [NO CHANGE]


(B) we workers must suffer despite
(C) us workers must suffer despite
(D) we workers must suffer due to

OK – first of all, we’re looking for pronouns, and one stands right out: “us.”

Who does “us” refer to (what’s the antecedent of the pronoun “us’)? “The workers.”

Is the pronoun “us” in the objective or subjective form? Check the chart: Objective.

Objective form means it’s not the subject, the main actor of the sentence, but the object -
having something done to it.

Now, think about the sentence as a whole. Does the sentence say anything is happening to the
workers? No, it says the workers are doing something: they are “suffering.”

If the workers are suffering, then they are the main characters of this sentence – they are the
subject. Therefore, any pronoun standing for “workers” in this sentence must be in the
Subjective Case.

Now, check the chart again for a reminder that “we” is the subjective form of “us.”

At this point we should check our answer choices: Choices A and C use the incorrect objective
form “us,” so they must be wrong.

Choice B and D seem almost identical at first, but notice the conjunctions they use: “despite” vs.
“due to.” A direct cause-and-effect is logical here, so the conjunction “due to” is the correct
choice over “despite.” If this confuses you, study Lesson 7 on Conjunctions. The correct answer
is Choice D.

Note: If this Subjective / Objective approach feels too technical for you, or you’re short on time
to prepare for your test, just stick with the previous method from earlier in this chapter:
simplifying/rearranging the sentence and trying each pronoun by itself, then relying on your ear
to help you. That will work for most questions, but it’s even better to understand this topic
from the technical angle as well.

88
Useful Links:
Redesigned SAT Practice Tests: http://www.cracksat.net/sat-tests/
SAT Online Practice Tests: http://www.cracksat.net/tests/
SAT Subjects Tests: http://www.cracksat.net/sat2/
SAT Downloads: http://www.cracksat.net/sat-downloads/
For more redesigned SAT information, please visit http://www.cracksat.net
SAT Grammar Practice Tests:

SAT Downloads:
Redesigned SAT Downloads: http://www.cracksat.net/sat-downloads/
SAT real tests download:
http://www.cracksat.net/sat-downloads/sat-real-tests.html
SAT official guide tests download:
http://www.cracksat.net/sat-downloads/sat-official-guide-tests.html
SAT online course tests download:
http://www.cracksat.net/sat-downloads/sat-online-course-tests.html
SAT subject tests download:
http://www.cracksat.net/sat-downloads/sat-subject-tests.html
PSAT real tests download:
http://www.cracksat.net/psat/download/

1000+ College Admission Essay Samples:


http://www.cracksat.net/college-admission/essays/
LESSON 7
Conjunctions & Transitions

The central concept of this lesson, titled “Conjunctions & Transitions,” is closely linked to
Conjunctions, one of the eight Parts of Speech you can review in Prelesson A.

The difference between those “conjunctions” (the Part of Speech) and “Conjunctions &
Transitions” (the SAT/ACT Grammar rule) is that this rule can apply either to the smaller level in
the middle of a single sentence (“Conjunctions”) or to the bigger level on a multiple-sentence
or multiple-paragraph level (“Transitions”).

Let’s take a look back at what a conjunction is:

A conjunction “joins” ideas A and B with a logical transition of some kind.

Common examples of conjunctions are and, however, but, or, because, although,
and many other “joining” or “transitioning” words.

“Conjunctions,” the name for this important part of speech, comes from the word “conjoined,”
or simply “joined.” A “junction” is a place where two things meet and connect – like two train
lines, two sides of a bridge, or two different ideas. This might help you remember: Conjunctions
are the words that join two ideas.

When two ideas are joined or connected, there are two possible situations: either the two ideas
are closely related, which I call “connectors,”, or they are in a disagreement of some kind, which
I call “divergers.”

Another way of saying this is that either the second idea continues or picks up where the first
idea left off (a “connector”), or the second idea will contradict the first idea (a “diverger”).

There are many different conjunctions, each with their own special “flavor” when it comes to
connecting or diverging between two ideas – but in the big picture, all transitions will fall
broadly into one of two categories: they either connect Idea B to Idea A, or they diverge from
Idea A to Idea B!

Conjunctions & Transitions also have two possible “strength levels”: strong and weak.

For example, the conjunction “because” is a strong connector that directly links Idea B to Idea A.
It shows a paired “cause and effect” - one of the strongest possible connections. On the other
hand, the conjunction “but” could be a weak diverger since it breaks a connection between two
ideas without necessarily implying a total contradiction of the first idea.

94
Examples of Common Conjunctions
Here are a few examples of common conjunctions:

Strong Connecting Conjunctions:


Because, since, due to, as a result

Weaker Connecting Conjunctions:


And, for, in addition, furthermore

Strong Diverging Conjunctions:


However, despite, nevertheless, not, instead

Weaker Diverging Conjunctions:


But, although, still, regardless

On the SAT & ACT grammar sections, Conjunction errors are often tested in sentences with no
obvious grammatical errors (for example, the Singular/Plural agreements are fine, the
pronouns play by the rules, no bad Parallelism or Misplaced Modifiers, etc.) but the answer
choices are focused on a selection of transitions or conjunctions - and in many cases, the
sentence’s meaning on the whole may make you shake your head in confusion and go “huh?”

In general, when the Coniunctions & Transitions are wrong, something about the big picture
won’t add up. The overall logic seems wrong - perhaps unclear - but maybe you can’t quite put
your finger on why. Look at your answer choices. Are the they playing with different
conjunctions and transition words?

These are all signs that you need to look for two separate ideas (“Idea A” and “Idea B”) and
closely examine the conjunction or transition options to connect them. Make sure the logical
pathway from Idea A to Idea B is expressed with maximum sense, logic, and clarity.

95
Let’s look at an example from the Pretest:

The soldier successfully pulled his ally from the helicopter wreck, and since
the flames engulfed their craft, he was unable to repair the vehicle.

(A) [NO CHANGE]


(B) helicopter wreck, the
(C) wreck of the helicopter, with the consequence that
(D) helicopter wreck, but because

Check out the basic logic of this sentence as written: In the first half, the soldier succeeds at
something (by saving his comrade). In the second half of the sentence, the soldier fails at
something else (he is unable to repair his helicopter due to flames).

Are those ideas connecting or diverging? Well, if he succeeded first, but failed second, I would
definitely have to say that these two ideas are diverging. If they were more connected, he
would either succeed in both halves or fail in both halves.

Unfortunately, the conjunction “and” is used to connect two ideas, so it’s illogical to use it in
this context. For a simpler illogical example, imagine:

“I usually can’t stand gym class and sometimes I like it”

This is illogical for the same reason as the “burning helicopter” Pretest sentence. The
conjunction “and” isn’t good for connecting two contrasting ideas. Instead of the connecting
word “and,” it will make more sense to use a contradicting conjunction like “but” or “yet.”

So, we know Choice A, “No Change,” is wrong. Choice B makes a run-on sentence (see Lesson 9
on Sentence Structure); we need to change the conjunction, but that doesn’t mean we can just
skip using any conjunction at all! We need to replace the connecting conjunction with a
contradicting one.

Choice C, “with the consequence that,” provides another connecting conjunction that implies a
direct cause-and-effect relationship between the two halves of the sentence. This is just as
wrong as using “and,” because it’s still not logical to say that “rescuing his ally” causes the
helicopter to be on fire.

Choice D uses “but,” which is the type of diverging conjunction we were looking for. Now the
soldier succeeds in the first half but fails in the second half - the sentence and its conjunctions
make sense.

96
Double Conjunctions
OK – there’s one other important way that the SAT & ACT hide conjunction errors. It’s less
common than what we just learned about, but still may make an appearance:

Important: Two ideas (“clauses”) cannot be connected by more than one conjunction.

This doesn’t mean an entire sentence is limited to just one conjunction (in fact, longer
sentences can often have more than one conjunction in them). It just means that two ideas
within the sentence must be connected by one conjunction and only one conjunction, whether
they are agreeing with or diverging from each other.

Here’s an example of a sentence that breaks this rule:

“Because I like fishing, since I’m very good at it.”

Notice the double conjunction “Because… since…” That’s not allowed when connecting two
ideas.

The Boxcar Method


Now, I want to teach you an advanced method I use myself to handle tricky transition questions
on the SAT & ACT. 99% of students never learn to do this, so they continually lose points to
challenging Conjunctions & Transitions questions. Hope you’re paying attention!

I call this method “the Boxcar method.”To get the idea, first picture a line of boxcars - train cars
- connected on both sides to other boxcars, forming an unbroken, connected chain of boxcars.
There are little hooks on both ends of every boxcar, used to hook together with the previous
and next boxcars in line so that the whole line of cars all connect.

Now, think of a Transition question as a set of three “idea boxcars” - each boxcar is one idea in
a three-idea sequence, and each idea hooks together on both sides, just like a chain of boxcars.
Our job is to transition from one idea to the next, so all three boxcars must hook together
smoothly and firmly on both sides: Boxcar A into Boxcar B into Boxcar C. Notice that Boxcar B,
in the middle, has the hardest job to do: it has to bridge between two connections, one on each
side.

So, to appropriately use the Boxcar Method, you should identify and focus on what I call
“Boxcar B” in the middle. Find this middle idea in the sequence of three ideas, and then split it
into “first half” and “second half,” which you should then summarize and write down on paper.

97
Now, use your summaries to eliminate any answer choices that do not link up the end of Boxcar
A to the first half of Boxcar B. By link up, I mean “show a logical connection.” (Remember,
you’re visualizing a line of connected “idea boxcars” from Boxcar A to Boxcar B to Boxcar C.)

Then, use your notes on the second half of Boxcar B to eliminate any options that don’t link up
the back end of Boxcar B to the start of Boxcar C.

Once you’ve patiently taken these methodical steps, it should become much easier to make
your final answer selection for any Conjunctions and Transitions question.

This is much faster and less work than it sounds, but only if you practice the method carefully
before the test. It was specially designed for SAT & ACT Transition & Conjunction questions, and
is extremely effective, although it takes practice to work efficiently.

Conjunctions & Transition Quick Reference


 A “junction” is where two things join, like a railroad junction. A “conjunction” is a
word that joins two ideas.

 “Conjunctions” connect two ideas within a sentence. “Transitions” connect larger


ideas between sentences or paragraphs.

 Identify these questions by their answer choices: you are given various choices for
conjunctions, connecting or transition words & phrases.

 There are two “flavors” of Conjunctions & Transitions: connecting and diverging.

 There are two “strengths” of Conjunctions & Transitions: stronger and weaker.

 Look at the “big picture” – the grammar may be fine, but does the logic make sense?
Does Idea A properly flow into Idea B?

 Use the “Boxcar Method” to narrow down to a single conjunction with the correct
“flavor” and “strength.”

 Only use one single conjunction between two ideas/clauses.

98
Note: We’ve been looking at Conjunctions that transition between halves of a single sentence -
but you should also be aware that the concept of Transitions and “boxcars” can be applied to
multi-sentence situations. Many questions on both the SAT and the ACT will demand that you
choose the optimal transition between two sentences, or even between two paragraphs.

The best way to approach these multi-sentence Transition questions is exactly the same as your
standard Conjunctions questions: Analyze the first half and the second half with the “Boxcar
Method,” taking written notes as you go. Next, consider if a “Connector” or a “Diverger” would
be more appropriate. Then eliminate your way down through the answers (don’t forget to
weigh “stronger” vs “weaker” transition options).

For now, we’ll continue to focus on Conjunctions and single-sentence situations. Once you’ve
mastered these situations and the “Boxcar Method,” you can use Official SAT and ACT practice
tests to identify and practice multi-sentence Transition questions.

Let’s look at another example from the Pretest:

Although deforestation is extremely detrimental to the planet’s delicate


ecology, yet many timber companies continue to harvest lumber for large
corporate profits.

(A) [NO CHANGE]


(B) and
(C) but
(D) [DELETE the underlined portion]

The two ideas (“Boxcars”) are: 1) Deforestation is harmful. 2) Many companies continue to
harvest lumber.

Is this a direct cause-and-effect relationship, or is it contradicting? Definitely contradicting. The


dangers of deforestation are contrasted with the continued timber harvesting. However, the
sentence is already using the conjunctions “although” and “yet,” both of which are diverging
conjunctions - so that seems good….

Wait a second - the sentence is using two conjunctions?! No way – only one single conjunction
is ever allowed to connect two ideas.

We can’t mess with the conjunction “although,” because it’s not underlined. The second
conjunction, “yet,” is underlined, so we should take this opportunity to remove it. Choice D is
the correct answer.

99
Useful Links:
Redesigned SAT Practice Tests: http://www.cracksat.net/sat-tests/
SAT Online Practice Tests: http://www.cracksat.net/tests/
SAT Subjects Tests: http://www.cracksat.net/sat2/
SAT Downloads: http://www.cracksat.net/sat-downloads/
For more redesigned SAT information, please visit http://www.cracksat.net
SAT Grammar Practice Tests:

SAT Downloads:
Redesigned SAT Downloads: http://www.cracksat.net/sat-downloads/
SAT real tests download:
http://www.cracksat.net/sat-downloads/sat-real-tests.html
SAT official guide tests download:
http://www.cracksat.net/sat-downloads/sat-official-guide-tests.html
SAT online course tests download:
http://www.cracksat.net/sat-downloads/sat-online-course-tests.html
SAT subject tests download:
http://www.cracksat.net/sat-downloads/sat-subject-tests.html
PSAT real tests download:
http://www.cracksat.net/psat/download/

1000+ College Admission Essay Samples:


http://www.cracksat.net/college-admission/essays/
LESSON 8
Punctuation Marks

There are 7 punctuation marks tested on the SAT & ACT tests:

Periods .
Commas ,
Semicolons ;
Colons :
Parentheses ()
Dash Marks -
Apostrophes ‘

You should not expect to see any questions on the SAT or ACT about question marks (?),
exclamation points (!), or quotation marks (“ “).

The majority of the tested punctuation marks - six out of seven of them - are used in connection
with clauses and Sentence Structure (the topic of the next lesson). The apostrophe is the only
punctuation mark tested on the SAT or ACT that does not relate to Sentence Structure.

Let’s go over the basics of each of these seven punctuation marks.

The Period
The period is our most familiar punctuation mark. Periods are used to end complete sentences.

Period Usage:
My dog is barking. My cat is watching birds.

That’s it - pretty simple. Also see the next lesson on Sentence Structure for more
information about Independent Clauses.

107
The Comma
The comma is the hardest punctuation mark for most students to master. Commas are not for
pauses or breaths as many students erroneously believe. Actually, Commas are used to
separate clauses (again, see the next chapter on Sentence Structure for related information).
Commas are also used to separate lists of three or more items.

Comma Usages:
My dog is barking, and my cat is watching him.
My dog, who is still a puppy, is barking.
My dog is barking, jumping, and running.

The Semicolon
Semicolons are very easy punctuation marks to master. The semicolon is used to separate two
complete sentences. On the SAT & ACT, semicolons work exactly like periods do. Don’t worry
right now about advanced nuances you may have heard before, such as “semicolons connect
two similar ideas.” Instead, just act like semicolons are periods, and you will always understand
their usage on the SAT and ACT.

Semicolon Usage:
My dog is barking; my cat is watching birds.

The Colon
Using colons is easy, but almost everyone misunderstands them. The most common
misunderstanding is the idea that “colons are for lists.” If this is what you think, you need to
change your definition - NOW.

Here’s how colons actually work. There are exactly two criteria.

The first criteria is crucial, but also easy and quick to identify, so check it first:

Colon Criteria #1: A colon must be preceded by a complete sentence.

The second criteria is what colons actually do:

Colon Criteria #2: “Colons introduce one or more examples, or a definition.”

108
Never, ever worry about the second colon criteria until after you’ve checked the first one.
Colons must always be preceded by a complete sentence. Knowing this one rule makes colon
usage 100x easier.

Colon Usages:
My house is filled with a variety of animals: dogs, cats, and birds.
I have a favorite animal: hedgehogs.
The musician played the lute: a classic instrument similar to an ancient guitar.

Parentheses
Parentheses are used to show optional, extra, or nonessential information that adds extra
detail to a sentence. You must be able to read the sentence without the parenthetical info and
still have the sentence work without it. Parentheses always come in pairs: an open parentheses
and a close parentheses.

Parentheses Usages:
My dog (a puppy) is barking.
I have two animals (a cat and a dog) who live at my house.

Notice that these examples can be read without the parenthetical info and still stand on their
own as complete sentences.

Dash Marks
Dash Marks can imitate either of two other punctuation marks: Parentheses or Colons. This is
determined by context.

When used as colons, dash marks follow exactly the same rules as colons. Easy!

However, dash marks are more often used like parentheses, with a few special notes:

First of all, when in the middle of a sentence, dash marks tend to come in pairs, just like
parentheses do. However, when used to add extra information at the beginning or end of a
sentence, a single dash mark is allowed to exist by itself. This would never allowed with
parentheses, which always have to be in an open-close pair.

109
Also, a bit of bonus info - though not essential for SAT/ACT testing - is that parentheses contain
extra information that is less important than the main sentence, while dash marks are meant to
contain info that is extra-important. Dashes call extra attention, while parentheses decrease the
amount of attention.

Dash Mark Usages:


My dog - still a puppy - is barking.
I have two animals - a cat and a dog - who live at my house.
I have a favorite animal - hedgehogs.

Apostrophes
These punctuation marks are a special case - apostrophes are the only marks on this list that do
not have anything to do with Sentence Structure (see the next lesson for more info).

Apostrophes mainly serve two purposes: to show possessive case for nouns and pronouns (for
example, Tim’s jacket or the birds’ cages), and to form contractions (shortened versions of a
pair of words).

Some students get mixed up when using the “plural possessive” form of a noun, which typically
places an apostrophe after the plural “s” at the end of a noun. For example, we might have four
birds in our house, living in four separate cages. In that case, to show that the birds “owned”
their cages, we’d put the apostrophe after the “s”, like so: “the birds’ four cages.”

Apostrophe Usages:
That is Tim’s jacket.
There are piles of seeds in the birds’ cages.
I can’t come to the door right now.
It’s time for dinner.

Note a special word in the last example above: its / it’s / its. One of these words doesn’t
actually exist!

Rules of its / it’s / its’:


it’s = Contraction of “it is”
Ex: It’s hot outside today.

its = Singular pronoun, possessive case


Ex: The cat was licking its tail.

its’ = This word does not exist in English!

110
Now let’s try a simple practice question from the Pretest.

There are several activities that I like to practice every day; motorcycling,
running, and songwriting.

(A) [NO CHANGE]


(B) every day: motorcycling,
(C) every day, motorcycling,
(D) every day. Motorcycling,

Since we are given a set of choices including semicolons, colons, and a periods, the first
question to ask is “can the first part of the sentence stand on its own?” (See Lesson 9 on
Sentence Structure for more on this topic.)

The answer is “yes” - the first half is a complete Independent Clause with its own subject and
verb. It could easily stand alone as its own sentence: “There are several activities that I try to
practice every day.”

The next question is, “What about the second half of the sentence? Can it stand on its own as a
complete, independent clause?”

The answer is no - the second half is not a complete sentence: “motorcycling, running, and
songwriting.”

We can now eliminate Choice D, which uses a period - that would require two complete
sentences, one on either side of the period. For the same reason, we can also eliminate Choice
A, “No Change,” because it uses a semicolon. Semicolons act exactly like periods and require a
complete sentence on both sides.

Can we use a comma? Unfortunately not, because there is no conjunction or connecting word
linking the first and second halves of the sentence. That means Choice C is eliminated.

Does a colon work? It might. We have a complete sentence before the colon (criteria #1). And,
the colon is followed by one or more examples (criteria #2). Perfect! This is the right moment to
use a colon, so Choice B is the correct answer.

111
Punctuation Marks Quick Reference
 There are seven punctuation marks to master for the SAT & ACT: periods, commas,
semicolons, colons, parentheses, dash marks, and apostrophes.

 Periods and semicolons come in between two complete sentences (also called
“Independent Clauses”).

 Colons have two criteria: a complete sentence before the colon, and one or more examples
or a definition after the colon.

 Dash marks can act like parentheses or colons, depending on context.

 Commas are challenging and require a mastery of clauses and Sentence Structure (see the
next lesson for more information).

 Apostrophes are different than the other punctuation marks, because they don’t usually
affect Sentence Structure. Instead, they are used for possessive case and contractions.

 Be well aware of the formal and technical differences between its / it’s / its’.

112
Let’s try another practice question.

There are many kinds of dogs, but mine - a golden retriever, is my favorite,
he is very loyal and friendly.

(A) [NO CHANGE]


(B) retriever - is my favorite; he
(C) retriever - is my favorite, he
(D) retriever) is my favorite; he

One of the best ways to get this Punctuation question started is to look for complete sentences
(or “Independent Clauses” - see the next chapter on Sentence Structure for more information).

We find an Independent Clause at the end of the sentence: “He is very loyal and friendly.” This
suggests that a period or semicolon must come just before this portion of the sentence, since
those are the two types of punctuation marks that can separate Independent Clauses.

We don’t have any answer choices with periods, but we do have Choices B and D, which both
use semicolons appropriately. So, let’s eliminate Choices A and C, which improperly use
commas to separate two Independent Clauses.

Now we can analyze the difference between Choices B and D. Choice D uses a close
parentheses “)” where Choice B uses a dash mark “-”. We can eliminate Choice D, because
there is no open parentheses in the sentence to pair with the close parentheses.

That leaves us with the correct answer, Choice B, which uses a pair of dash marks to isolate a
Parenthetical Clause (“a golden retriever”) and also uses a semicolon to separate two
Independent Clauses from each other.

We’ll continue to learn more about the challenges of commas and other punctuation marks in
the next chapter, because punctuation is closely tied with the upcoming lesson on clauses and
Sentence Structure.

113
Useful Links:
Redesigned SAT Practice Tests: http://www.cracksat.net/sat-tests/
SAT Online Practice Tests: http://www.cracksat.net/tests/
SAT Subjects Tests: http://www.cracksat.net/sat2/
SAT Downloads: http://www.cracksat.net/sat-downloads/
For more redesigned SAT information, please visit http://www.cracksat.net
SAT Grammar Practice Tests:

SAT Downloads:
Redesigned SAT Downloads: http://www.cracksat.net/sat-downloads/
SAT real tests download:
http://www.cracksat.net/sat-downloads/sat-real-tests.html
SAT official guide tests download:
http://www.cracksat.net/sat-downloads/sat-official-guide-tests.html
SAT online course tests download:
http://www.cracksat.net/sat-downloads/sat-online-course-tests.html
SAT subject tests download:
http://www.cracksat.net/sat-downloads/sat-subject-tests.html
PSAT real tests download:
http://www.cracksat.net/psat/download/

1000+ College Admission Essay Samples:


http://www.cracksat.net/college-admission/essays/
LESSON 9
Sentence Structure

Sentence Structure is a grammar topic that is intimately tied with Punctuation Marks (Lesson 8),
because one of the main uses of punctuation marks is to support the “structure” of a sentence
(and you’ll learn more about what exactly this means in just a moment).

I have often noticed that high school students struggle to master Sentence Structure, but it’s
worth a lot of points on the SAT & ACT, so we should strive to understand the rules of good
Sentence Structure.

What is Sentence Structure?


What do I mean by “Sentence Structure”? Specifically, I refer to the way a sentence is built out
of smaller phrases or “clauses” and how those clauses fit together to form a grammatically-
acceptable sentence.

So, before we master Sentence Structure we will need to go over the three types of clauses. I
will make it as painless and quick as possible!

A “clause” is defined as “a unit of grammatical organization just below the sentence in rank.”
That basically means any group of words working together in a single “phrase.” There are three
types of clauses:

1. Independent Clause: A complete idea that can stand on its own. It contains a Subject
and a Main Verb, but not a subordinating word or conjunction (words like and, because,
so, or, but, etc.)

2. Dependent Clause: A “Dependent Clause” is essentially an “Independent Clause” with


a subordinating word (e.g. and, because, so, or, etc.) attached to it. This creates an idea
that cannot stand on its own, and must be attached to an Independent Clause to survive.

3. Parenthetical Clause: A “Parenthetical Clause” does not need its own Subject-Verb
pair (a topic explored in Lesson 1). Instead, it contains supporting details - the kind we
could enclose in parentheses (thus the name “Parenthetical” clauses). Parenthetical
Clauses can always be completely removed from sentences without harming the
underlying basic meaning or structure.

120
Examples of the Three Types of Clauses

Here is a set of examples showing all three types of clauses:

1. The dog barked.


Above is an Independent Clause with “dog” as the subject and “barked” as the main verb.

2. The dog, a golden retriever, barked.


We’ve added a Parenthetical Clause inside of the Independent Clause, marking the separation
with commas.

3. The dog barked, but his owner did nothing.


We’ve taken the Parenthetical Clause back out and instead linked a Dependent Clause to the
Independent Clause.

4. The dog barked, but his owner, my neighbor, did nothing.


We’ve added a Parenthetical Clause inside of the Dependent Clause.

5. The dog, a golden retriever, barked, but his owner, my neighbor, did nothing.

All together now: the first half of the above sentence is an Independent Clause (“The dog
barked”) with a Parenthetical Clause inside it; the second half of the sentence is a Dependent
Clause (“but his owner did nothing”) with its own Parenthetical Clause inside it, too.

Next up, some more details and finer points about the three types of clauses…

Independent Clauses
Every Independent Clause has a subject. The subject will always be a Noun, Pronoun, or Gerund
(a verb that’s been made into a noun, like running in the sentence “Running is fun.”) Since verbs
are “doing” or “being” words, the subject is the primary “do-er” of action or the “be-er” that is
being described.

Every Independent Clause also has a main verb, the action that the subject is doing (“The dog
runs,” “The boys go to school”), or a “being” verb (as in “The cat is happy,” “Our old house was
painted red,” or “The children are playing”).

121
Finally, every Independent Clause expresses a complete idea that can stand on its own. “The cat
is happy” can stand on its own. (On the other hand, the clause “because the cat is happy”
cannot stand alone - the subordinating word “because” makes it a Dependent Clause.)

That’s it! All you need for a complete Independent Clause is a subject and a main verb that add
up to one complete idea.

Dependent Clauses
Independent Clauses may provide the most essential structure to our sentences, but Dependent
Clauses are also useful. These clauses usually have a subject and verb pair, but can’t quite stand
on their own because they also contain a subordinating word.

Subordinating words include many conjunctions (such as and, but, because, or, and not). Some
of these conjunctions are part of the “FANBOYS” family (for, and, nor, but, or, yet, so). These
are sometimes called “coordinating conjunctions.”

An example of a Dependent Clause that cannot stand on its own is “because I am happy.”
Notice the subordinating word “because.”

Without an Independent Clause to attach to (such as “I am smiling”), this Dependent Clause


CANNOT stand alone!

We could create a combined sentence with an Independent and Dependent Clause such as “I
am smiling, because I am happy.”

Parenthetical Clauses
Parenthetical Clauses are frequently used to add additional details or explanation. They’re
definitely important - they’re just not as important as Independent or Dependent Clauses.

For example, we’ve already seen the Parenthetical Clause “a golden retriever” used to modify a
“dog,” as in the example “The dog, a golden retriever, barked.” You can see how the
Parenthetical Clause is adding interesting, but non-essential detail, to the Independent Clause.
You might also observe that you could us parentheses - (a golden retriever) - instead of commas.

Parenthetical Clauses are frequently introduced by an “-ing” word, as in “The dog, barking
loudly, jumped at the fence.” Other common introductions to a Parenthetical Clause are certain
subordinating words like who, where, or which - these are called “Relative Pronouns” - as in
“The dog, who barked loudly, jumped at the fence.”

Parenthetical Clauses can be completely removed from a sentence without affecting the
essential meaning or structure of the sentence.

122
Identifying Different Types of Clauses
Part of your job is to get good at telling which types of the three clauses you’re dealing with.
One way to make that easier is to use Eliminating Details (Prelesson B) to trim down the extra
details in the sentence.

Example 1:
Let’s try analyzing the following sentence:

“The brown bear jumped from tree to tree.”

What are the subject and main verb?

Subject: The bear (If you said “brown” bear, notice that “brown” is just an adjective that gives
more details and is not as crucial as the subject noun, “bear.”)

Main Verb: Jumped (the action word that the subject is doing)

We can ignore the prepositional phrase “from tree to tree,” which simply adds detail about the
location of the bear. Prepositional phrases can always be ignored when analyzing clauses (you
can review Prelesson A on Parts of Speech for more info about prepositions.)

Can this idea (“The bear jumped”) stand on its own? Sure, it’s a complete thought. This is an
Independent Clause and it stands on its own just fine.

Example 2:
What about this example?

“But the roller coaster is fun.”

What are the subject and main verb?

Subject: The roller coaster

Main Verb: Is

Can this idea stand on its own? No! This clause says “but the roller coaster is fun.”

“But” what, exactly?

123
This thought seems to be contradicting some other idea, but it can’t stand alone. This clause
needs to connect to something to be complete! The word but is an example of a subordinating
word (it’s a FANBOYS coordinating conjunction), which turns this phrase into a Dependent
Clause.

We could fix it by connecting it to a complete idea:

“The merry-go-round is boring, but the roller coaster is fun.”

OK, so that example was a little simplistic. The way the SAT & ACT make clause analysis more
difficult is by adding a heap of extra unessential details into the sentence in the form of
adjectives, adverbs, prepositional phrases, and Parenthetical Clauses.

These unessential details often arrive with a bunch of commas surrounding them, and act like
camouflage for grammatical problems hidden below the surface. For example, once you
remove the details, you may find only a collection of Dependent Clauses with no strong, heroic
Independent Clause to support them all. You may even find a “Sentence Fragment,” or
incomplete Independent Clause.

In other words, eliminating extra details is a trick you will use throughout the SAT & ACT
Grammar sections on nearly every single question. If you’re not already a pro at eliminating
details, re-read Prelesson B on Eliminating Details!

Example 3:
Let’s take a look at another example and analyze the clauses:

“The duck, who is eating pancakes, likes breakfast, and he also enjoys pickles.”

First we should look for an Independent Clause. Where is the subject noun of this sentence? (If
you said “duck,” you are correct.)

Now let’s find the main verb. It’s a little tricky - the main verb is likes, not “eating.” The
complete Independent Clause is “The duck likes breakfast.”

This Independent Clause is interrupted by a Parenthetical Clause: “who is eating pancakes.”


Notice that this Parenthetical Clause is introduced by the relative pronoun who.

There is also a Dependent Clause at the end of the sentence: “and he also enjoys pickles.”
Observe that this Dependent Clause is essentially an Independent Clause (including the subject
“he” and main verb “enjoys”) paired with the subordinating FANBOYS conjunction “and”.

124
Example 4:
Here’s an example where you should Eliminate Details to make the main clauses more obvious:

“The brown bear, which is known for eating various foods left behind by
picnickers, who visit national parks on the weekends.”

What are the unessential descriptive details in this sentence? Remove them first:

1) “which is known for eating various foods” is a big Parenthetical Clause that just
describes the bear. (Note that this clause is introduced by the Relative Pronoun
“which.”)

2) “left behind by picnickers” is a prepositional phrase that simply describes the


location of the foods.

3) “who visit national parks on the weekends” is a Parenthetical Clause that


describes the picnickers (notice that the clause is introduced by the Relative
Pronoun “who.”)

Take descriptive phrases 1, 2, and 3 out, and suddenly the camouflage of the sentence is gone:
we’re left holding a sentence with nothing left in it besides “The brown bear.”

Where’s the main verb? What “is” the bear, or what does it do? Absolutely NOTHING! The main
verb is missing in action; we’re left holding only a subject with no main verb. This is called a
“Sentence Fragment,” and this incomplete sentence is a huge Sentence Structure no-no.

Using Punctuation Marks for Better Sentence Structure


It’s important to understand that Punctuation Marks (see the previous lesson) and Sentence
Structure work hand-in-hand to produce good grammatical results.

Remember and apply the following rules:

1. Periods end complete sentences.

2. Semicolons are very similar to periods and go between two Independent Clauses.

3. Colons must be preceded by an Independent Clause and followed by one or more examples
or a definition.

4. Commas are used to separate clauses. For example, a comma could separate an Independent
Clause from a Dependent Clause, or separate a Parenthetical Clause from an Independent or
Dependent Clause. There is one important exception: commas cannot be used between two
Independent Clauses, or you will have a “Comma Splice” error (see below).

125
Comma Splices
A “Comma Splice” error will occur if you try to connect two Independent Clauses with only a
comma. For example:

“James threw his fork, he was not happy with his meal.”

This sentence contains two Independent Clauses (the first half and the second half). But, we
cannot simply use a comma to connect these two Independent Clauses, because that creates a
Comma Splice error.

The three main options for fixing this mistake include:

1) Use a period between the two Independent Clauses:

“James threw his fork. He was not happy with his meal.”

2) Use a semicolon between two Independent Clauses:

“James threw his fork; he was not happy with his meal.”

3) Use a comma + subordinating conjunction to change one of the Independent Clauses into a
Dependent Clause:

“James threw his fork, because he was not happy with his meal.”

126
Sentence Structure Quick Reference
 Every sentence needs at least one Independent Clause. Without an Independent Clause, we
have a Sentence Fragment error.

 An Independent Clause requires two things: a subject and a main verb. It should also express a
complete idea that can stand alone.

 Independent Clauses can end with a period, a semicolon, a comma + Dependent Clause, or a
colon used properly according to the rules of colon usage (review the previous chapter on
Punctuation Marks).

 Dependent Clauses have a subject and verb but not a complete thought; they use a
subordinating conjunction (like but, because, or yet) and must attach to an Independent Clause
via a comma.

 Parenthetical Clauses contain supporting details that could be enclosed in parentheses. You can
always ignore these clauses without changing the sentence’s essential meaning or structure.
These are frequently separated from other clauses by commas, parentheses, or dash marks.

 Two Independent Clauses cannot be connected with only a comma or you will have a Comma
Splice error. You could use a period or a semicolon. If you choose to use a comma, you must
change one of the two Independent Clauses into a Dependent Clause by adding a subordinating
conjunction like “because” or “so.”

 Eliminating Details (Prelesson B) - especially adjectives, adverbs, prepositional phrases, and


Parenthetical Clauses - is a good way to simplify a sentence and expose hidden errors in its
Sentence Structure.

127
Let’s take a look at a question from the Pretest:

The “desert rally,” an endurance race by dune buggies across hostile


terrain, that only the fiercest competitors qualify for.

(A) [NO CHANGE]


(B) which only the fiercest competitors can qualify for.
(C) and only the fiercest competitors qualify for it.
(D) is a contest that only the fiercest competitors qualify for.

Take a look at the original sentence. We’re going to practice Prelesson B: Eliminating Details.
Find and remove all the unessential details to make the underlying Sentence Structure more
clear.

First, “an endurance race by dune buggies across hostile terrain” is a nonessential Parenthetical
Clause adding details about the “desert rally.” We can remove it without affecting the
underlying sentence.

Second, “that only the fiercest competitors qualify for” is a descriptive phrase that adds
additional details about the “desert rally.” It also can be removed.

Take out phrases 1 and 2 and you’re left with only “The desert rally,” which is the subject of this
clause. But where’s the main verb? It’s nowhere to be found, which means this is a Sentence
Fragment error!

Repeat this analysis with Choices B and C and you’ll notice the same problem.

Choice D adds the main verb is. Phew – now the desert rally “is” something! The new &
improved sentence has a subject, a main verb, and expresses a complete thought. That’s good.
Now we have a complete Independent Clause, and Choice D is the right answer.

128
Let’s check out another example from the Pretest:

The intrepid explorers of the wilderness discovered a new river, they also
found a pair of mountains they named “Dipyramid.”

(A) [NO CHANGE]


(B) river, found
(C) river; they also find
(D) river and found

In the original version, notice the comma hanging out in the middle - without a subordinating
word, such as a FANBOYS conjunction, to create a Dependent Clause? Uh-oh, that’s often a bad
sign for Sentence Structure problems.

First off, are there any Independent Clauses ? Here’s where simplification can come in handy.
We can use Eliminating Details (Prelesson B) to reduce this sentence to the following:

“The explorers discovered a river, they also found mountains.”

Yes - unfortunately, we do have two complete Independent Clauses joined only by a weak
comma. It’s a Comma Splice error.

Choice C might seem like a logical pick, with the semicolon between two Independent Clauses,
but it creates a Verb Tense error by changing the past-tense “found” to a present-tense “find.”
This creates a mismatch between the tenses of the first and second half of the sentence. If you
rush through this question, it’s easy to overlook this error. Don’t rush!

What other option do we have, besides a semicolon? Well, the two clauses in the sentence
connect logically – they both describe things that the explorers discovered. A simple,
connecting FANBOYS conjunction such as “and” would work nicely:

“The explorers discovered a river and found mountains.”

That’s why Choice D is perfect for this situation, and the Comma Splice error is fixed. Choice B
almost works (because the second half of the sentence loses its subject, “they,” and so isn’t an
Independent Clause any longer), but it would need a coordinating conjunction after the comma.

129
Here’s another Pretest question:

Authors selected for the philosophy journal must be respected in their field,
and should hold advanced degrees and have a great deal of practical
experience.

(A) [NO CHANGE]


(B) his or her field, and
(C) their field, they
(D) their field, such authors

Let’s Eliminate Details (Prelesson B) and simplify the sentence to analyze the clauses more
easily:

“Authors must be respected, and should hold degrees.”

Right now we have an Independent Clause and a Dependent Clause connected by a comma and
conjunction (“and’), a textbook example of good Sentence Structure. Let’s look at our other
choices, just to be sure.

Choice B maintains good Sentence Structure, but unfortunately adds a Pronoun-Antecedent


Agreement (Lesson 5) error: the singular pronoun “his or her” doesn’t match with the plural
antecedent “authors.”

Choices D creates a Comma Splice error: there is an independent Subject-Verb pair on both
sides of the comma (“Authors must” and “authors should hold”). This creates two Independent
Clauses connected by only a comma with no conjunction - exactly the type of error we must
avoid.

Choice C has the same problem, because it simply replaces the noun “such authors” with the
pronoun “they,” which doesn’t fix the underlying problem of a Comma Splice created by an
improperly-punctuated pair of Independent Clauses.

Only Choice A prevents a Comma Splice by using a comma paired with the subordinating
FANBOYS conjunction “and.”

130
Let’s do one more example from the Pretest:

The pine tree, known for its majestic appearance and sweet scent, and its
needles can be used to brew tea.

(A) [NO CHANGE]


(B) The pine tree, known for its majestic appearance and sweet scent, has
needles that can be used to brew tea.
(C) Known for its majestic appearance and sweet scent, the pine tree’s
needles can be used to brew tea.
(D) The pine tree, its needles can be used to brew tea, is known for its
majestic appearance and sweet scent.

Let’s analyze for clauses and Sentence Structure. Starting with the original sentence, eliminate
any camouflaging unessential details. First, the Parenthetical Clause “known for its majestic
appearance and sweet scent” can be removed without affecting the underlying sentence.

Now the sentence is just “The pine tree and its needles can be used to brew tea.”

This sentence contains a logical error that’s camouflaged by its structure: you would not use a
pine tree and its needles to brew tea - you would only use the needles. Can you imagine the
ridiculousness of trying to brew tea with an entire tree? This is an example of how Sentence
Structure can be used to hide critical underlying mistakes in the meaning of the sentence.

Choice C is wrong not because of Sentence Structure, but because of a combined Pronoun-
Antecedent Agreement (Lesson 5) and Misplaced Modifier error (Lesson 11) that incorrectly
uses the singular pronoun “its” to refer to the plural antecedent “needles” and also makes it
seem like the needles are “known for their majestic appearance,” when really the tree would be
more likely be known for its majestic appearance.

Choice D is wrong because of a Comma Splice error. Break the sentence into three sections: the
first (“The pine tree”) provides a subject, which is paired with a main verb at the end of the
sentence (“is known”) - so, that’s one Independent Clause already. Unfortunately, the middle
section provides a second Independent Clause with the Subject-Verb pair “needles can be
used.” The two Independent Clauses are mashed together with only commas to connect them.
No periods, no semicolons, no subordinating words… not an acceptable way to join two
Independent Clauses.

Choice B is correct. Removing all details, including the central Parenthetical Clause “known for
its majestic appearance and sweet scent” as well as the descriptive phrase “that can be used to
brew tea,” leaves behind a single Independent Clause: “The pine tree has needles.” Our
meaning is also clear and accurate. Finally, we’ve achieved good Sentence Structure!

131
Useful Links:
Redesigned SAT Practice Tests: http://www.cracksat.net/sat-tests/
SAT Online Practice Tests: http://www.cracksat.net/tests/
SAT Subjects Tests: http://www.cracksat.net/sat2/
SAT Downloads: http://www.cracksat.net/sat-downloads/
For more redesigned SAT information, please visit http://www.cracksat.net
SAT Grammar Practice Tests:

SAT Downloads:
Redesigned SAT Downloads: http://www.cracksat.net/sat-downloads/
SAT real tests download:
http://www.cracksat.net/sat-downloads/sat-real-tests.html
SAT official guide tests download:
http://www.cracksat.net/sat-downloads/sat-official-guide-tests.html
SAT online course tests download:
http://www.cracksat.net/sat-downloads/sat-online-course-tests.html
SAT subject tests download:
http://www.cracksat.net/sat-downloads/sat-subject-tests.html
PSAT real tests download:
http://www.cracksat.net/psat/download/

1000+ College Admission Essay Samples:


http://www.cracksat.net/college-admission/essays/
LESSON 10
Parallelism & Comparisons

When’s the last time you thought about the word “parallel” outside of math class? Maybe
never, so this could be a first for you!

In math class, “parallel” lines could be described as lines that are going exactly the same
direction as one another. They’ll never bump in to each other, because they’re in perfect
alignment.

Believe it or not, grammar has “parallel” situations, just like math – and the concept is similar.

In math you must have at least two lines for them to be parallel (a single line, by itself, isn’t
really parallel to anything, is it?). However, you don’t have to stop with two lines; you can have
as many different parallel lines as you want, as long as they’re all going in exactly the same
direction.

The same holds true in grammar. “Parallelism” issues come up in sentences involving lists or
comparisons of two or more things.

There are three main situations when Parallelism will happen:

1) In a simple list (like a grocery list).


2) Between two or more longer phrases.
3) Within a comparison.

Parallelism Type #1 is the simplest style, so let’s look at it first…

Parallelism Type #1: “Grocery Lists”


One extremely common example of Parallelism would be a list, such as the following:

Example of a good parallel list:

“I went to the store for eggs, milk, and cheese.”


The list doesn’t have to actually be a grocery list, but it’s one of the easiest examples to
remember (a non-grocery list example could be “I like running, jumping, and swimming”).

We have three items in this grocery list, all of which are nouns (reviewing your Parts of Speech
from Prelesson A is very helpful when studying Parallelism). That’s good; the list follows proper
parallel structure because all three items in the list are the same Parts of Speech. In our math

143
analogy, three nouns would be like three lines that all go the same direction – they are parallel
and don’t bump into each other, which is good.

Where we’d have a Parallelism problem would be if we mixed and matched nouns and some
other parts of speech in our list, like so:

Parallelism Mistake Version:

“I went to the store for eggs, milk, and to buy cheese.”


Now one of our grocery list items has changed direction, grammatically speaking – the noun
“cheese” has changed into “to buy cheese,” a phrase which includes a verb. This breaks the
pattern and commits a Parallelism error. All items in a list of two or more things must be the
same Parts of Speech in the same forms.

When it comes to making a list or comparison of two or more items, nouns match only with
other nouns. Verbs match only with other verbs of the same form. Words ending in “-ing”
match only with other “-ing” words (running, jumping, swimming.) None of these categories
can be mixed and matched within the same list or comparison, or it will cause a Parallelism
error.

Here are some BAD examples of grocery lists that break parallel form:

“I went to the store for eggs, milk, and buying cheese.”


“I went to the store for eggs, to buy milk, and bought cheese.”
“I went to the store for buying eggs, to get milk, and cheese.”
“I went to the store and bought eggs, was getting milk, and cheese.”

Here are some GOOD examples of grocery lists that follow parallel form:

“I went to the store for eggs, milk, and cheese.” (Three nouns)
“I went to the store to buy eggs, to find milk, and to sample cheese.” (All three
items follow the same pattern: “to [verb] [noun]”

Let’s look at an example from the Pretest:

The bass guitar differs from standard guitars in that the bass has fewer
strings, a longer neck, and the ability to produce lower notes.

(A) [NO CHANGE]


(B) has fewer strings, a longer neck, and can produce lower notes.
(C) having fewer strings, a longer neck, and can produce notes more low.
(D) with lower strings, a longer neck, and lower notes.

144
First, take a look at Choice B and notice the Parallelism mistake: The bass has a list with three
items in it: “fewer strings” (a noun), “a longer neck” (a noun), and “can produce lower notes”
(“can produce” is a verb!)

We need the third item in this list to be another noun so that the whole list is nice and parallel.

Here’s another way to look at this error - break the list in Choice B down into three traits of the
bass guitar and test one at a time to expose the mistake:

1) The bass has fewer strings. (check!)


2) The bass has a longer neck. (check!)
3) The bass has can produce lower notes. (uh… what?!?)

Choices B and C both contain the same Parallelism mistake. Choice D fixes the Parallelism
problem, but removes the main verb “has.” Now the subject “the bass” no longer has a main
verb attached to it, creating a Sentence Fragment (review Lesson 9 on Sentence Structure for
more details).

Choice A is correct; compared to Choice B, it changes the third item in the list, “can produce
lower notes,” to “the ability to produce lower notes.” “Ability” is a noun, so now the list is
made up of three nouns: “strings, neck, and ability,” and the law of Parallelism is satisfied.

Parallelism Type #2: Parallelism in Longer Phrases


This is the “big brother” of Type 1 Parallelism (“grocery lists”), and requires a little more care
and attention. Most people can easily recognize grocery-style lists of three simple items, but it’s
harder to notice a “list” that is made up of larger phrases, rather than individual words. Often,
these sentences will only have two items in the list.

Here’s an example of a Parallelism mistake of this type:

“Would you prefer to be eaten by an alligator or getting digested by a shark?”

The “list” in this sentence is only two items long: One, “to be eaten by an alligator,” or two,
“getting digested by a shark.”

The first step is to recognize the list of two items and identify the risk of a Parallelism mistake.
The second step is to investigate the parallel structure: are “to be eaten” and “getting digested”
the same Parts of Speech in the same form as each other? The answer is no: although both
items are both based on verbs, the forms of the verbs are different (“to be x-ed” vs “x-ing”).

145
If you want to fix the Parallelism of this sentence, you have two options:

Option A) “to be eaten” and “to be digested”

Option B) “getting eaten” and “getting digested”

But we can’t mix and match “to be” and “getting” because they’re verbs in different forms.

This kind of Parallelism error can definitely be tricky to notice. The biggest giveaway for this
kind of mistake is to look for conjunctions such as “and,” “or,” and “not only.. but also.” When
listing or comparing things, the items you’re listing or comparing should be the same Parts of
Speech in the same forms.

It’s easier to see what I mean with some examples. The words in italics are the things that must
be parallel to each other, while the words in bold are the giveaway connecting/comparing
words:

Wrong: “Going to the rodeo is more fun than to go biking.” (“Going” and “to go” are
different verb forms)

Right: “Going to the rodeo is more fun than going biking.”

Wrong: “My brother likes not only his life in the dorm, but also to jump on his bunk
bed.” (“his life” is a noun, but “to jump” is a verb.)

Right: “My brother likes not only living in his dorm, but also jumping on his bunk bed.”

Here’s another example from the Pretest:

Beethoven’s music is known for being logical, yet emotive; challenging, yet
occasionally simple; and unique, yet derived from previous styles.

(A) [NO CHANGE]


(B) previous styles influenced it.
(C) was derived from previous styles.
(D) it was derived from previous styles.

Here we have a complex list of three things that Beethoven’s music is known for, with the first
two items following the pattern “adjective, yet other contrasting adjective.”

It’s the last item in the list that we’re particularly interested in, because it’s underlined. Does it
break the pattern?

146
Well, let’s look at our options. Choice D breaks the Parallelism pattern with “it was derived”
(pronoun “it” + verb “was derived”). These new Parts of Speech are suddenly introduced into
the list, conflicting with the Parallelism pattern of “adjective, yet other contrasting adjective”
that was already established by the first two items of the list.

Similar Parallelism problems crop up in Choice B (which contains an unwelcome verb,


“influenced,” that doesn’t fit into the established pattern) and Choice C (which also contains a
verb, “derived”).

The best choice is to maintain the Parallelism pattern that’s already established - use Choice A,
which keeps the last part of the sentence as a short modifying phrase, “derived from previous
styles.”

A modifying phrase (like “occasionally simple,” or “derived from previous styles”) can act the
same way as a single adjective (like “emotive.”) This makes sense if you think about it, because
modifying phrases are like extended versions of single adjectives – both share the function of
adding description or detail to a target noun.

Be aware that “modifying phrase” is simply another name for “Parenthetical Clauses,” which
you can review in-depth with Lesson 9 on Sentence Structure!

Side Note: Using Semicolons in Lists


If you’re wondering about our unusual usage of semicolons in the previous example, did you
know that semicolons can be used to punctuate lists?

Usually we use commas to punctuate items in a list:

“I need eggs, milk, and chicken.”

However, for complex lists we can use semicolons as separators, as when you make a list of
cities:

“I’ve visited Austin, TX; Buffalo, NY; and Los Angeles, CA.”

See how we can use semicolons to punctuate complex lists - especially if the items in the list
already have commas within them? That’s the same technique we use in the “Beethoven”
Pretest example above.

147
Parallelism Type #3 - Parallelism in Comparisons
When any sort of comparison is made between two or more items, those items must be
logically comparable and follow the rule of Parallelism.

It’s easy to demonstrate. Here are some items that CAN be properly compared:

Cats vs. Dogs (they are both animals)

Apples vs. Oranges (they are both fruit)

Firemen vs. Policemen (they are both professions/types of people)

Speed of a car vs. Speed of a motorcycle (they are both velocities)

Now, here are some items that could NOT be properly compared:

Cat hair vs. Dogs (we can’t compare “hair” to an animal like a “dog”; it’s not a
logical comparison to make; change it to cat hair vs. dog hair)

Apples vs. Orange Trees (one is the fruit, the other is the tree it grows on)

Firemen’s salaries vs. Policemen (one is money, the other is a person/profession)

A car vs. speed of a motorcycle (one is a vehicle, the other is a velocity)

The Parts of Speech must still match in your comparisons, as with all Parallelism situations.
Here is an example of what happens when this rule is broken within a comparison:

BAD Comparison / Parallelism: I like to run more than jumping.

The example above is wrong because it uses different verb forms and breaks Parallelism. You
could fix it either by pairing “running” and “jumping,” or by pairing “to run” with “to jump.” Just
no mixing-and-matching of forms.

One tricky Comparison variation. Does the following sentence sound wrong to you?

“The length of this train is more impressive than any bus I’ve ever seen.”

This may sound acceptable, but it is wrong. Take a look at exactly what’s being compared:

“Length of this train” vs. “any bus.” It’s illogical to compare a length to a vehicle. Fix the
sentence by changing to “the length of any bus.”

As you train yourself to notice this error, watch out for any kind of “comparing” word; this can
range from “more” to “bigger,” “wiser,” “louder,” etc.

148
In fact, the single biggest giveaway that this Parallelism / Comparison error might be under
your nose is the word “than” - a word that always indicates a comparison.

Parallelism & Comparisons Quick Reference:


 In math, “parallel” lines are when two or more lines are all going the exact same direction
as each other - not bumping into one another.

 In grammar, words in a comparison or list of two or more items are all “going the exact
same direction” if they have the same Parts of Speech in the same forms as each other.

 Simply stated, the rule of Parallelism is: “In any list or comparison, all items must be the
same parts of speech in the same forms.”

 Comparisons must always be between items that are logically comparable to each other. In
addition, all Comparisons must follow the rule of Parallelism.

 Train yourself to watch out for lists and comparisons and double-check that they are
consistent all the way through.

 The most common giveaway for a comparison is the word “than.” Words that end in “-er”
(like “more” or “taller”) are another way to notice comparisons.

 The harder Parallelism questions may present two longer phrases linked by a conjunction.
Both phrases should be formed as similarly as possible to one another, using the same Parts
of Speech in the same forms.

149
Let’s look at a Parallelism / Comparison example from the Pretest:

The use of robots on farms is growing rapidly, since it is often more cost-
effective than human farm workers.

(A) [NO CHANGE]


(B) more cost-effective than the use of
(C) more effective than using
(D) less costly than

Notice the words “more cost-effective than,” which indicate a comparison.

What two things are being compared to each other? “The use of robots” vs. “human farm
workers.” This is no good; use is not parallel to human worker.

The two ways to fix this would be to compare “use of robots” to “use of human workers,” or
compare the nouns “robots” to “workers,” but regardless of which you choose, it’s incorrect to
mix-and-match. Choice B is the only option that properly compares “the use of robots” to “the
use of human farm workers,” maintaining the law of Parallelism.

Choice C and D continue the original Parallelism / Comparison mistake by comparing “use” to
“using” in Choice C (different Verb Forms, see Lesson 2) or comparing “use” to “human
workers” as in the original mistake.

150
Here’s a final Pretest question:

I think that most Japanese manufacturers’ cars get better mileage and are
more reliable than American automakers.

(A) [NO CHANGE]


(B) making cars in America
(C) American automakers’ cars
(D) what they make in America

Notice now how this sentence is trying to compare the cars from Japan to the auto makers
from America? You know by now that this breaks the rules of Parallelism & Comparison, so go
with Choice C, which correctly compares cars made by Japanese manufacturers to cars made by
American automakers.

Go forth and conquer in your Comparisons & Parallelism! It will earn you a ton of extra points on
your grammar sections. This is a really simple topic that you just have to train yourself to
become aware of - I promise you’ll see it often on the SAT & ACT tests, once you start actively
looking for it!

151
Useful Links:
Redesigned SAT Practice Tests: http://www.cracksat.net/sat-tests/
SAT Online Practice Tests: http://www.cracksat.net/tests/
SAT Subjects Tests: http://www.cracksat.net/sat2/
SAT Downloads: http://www.cracksat.net/sat-downloads/
For more redesigned SAT information, please visit http://www.cracksat.net
SAT Grammar Practice Tests:

SAT Downloads:
Redesigned SAT Downloads: http://www.cracksat.net/sat-downloads/
SAT real tests download:
http://www.cracksat.net/sat-downloads/sat-real-tests.html
SAT official guide tests download:
http://www.cracksat.net/sat-downloads/sat-official-guide-tests.html
SAT online course tests download:
http://www.cracksat.net/sat-downloads/sat-online-course-tests.html
SAT subject tests download:
http://www.cracksat.net/sat-downloads/sat-subject-tests.html
PSAT real tests download:
http://www.cracksat.net/psat/download/

1000+ College Admission Essay Samples:


http://www.cracksat.net/college-admission/essays/
LESSON 11
Misplaced Modifiers

Misplaced Modifier errors are one of my favorite mistakes in the SAT & ACT Grammar sections.
In a test that is almost entirely boring and humorless, this type of mistake can bring a little bit
of comedy to your experience. You just have to know how to appreciate it…

What is a “Modifier”?
A “modifier” or “modifying phrase” is essentially another name for a “Parenthetical Clause”
(review Lesson 9 on Sentence Structure for more details.) These phrases are never essential to
the main sentence; they simply add extra details about other elements of the sentence.

Here’s an example of a “modifying phrase” (or Parenthetical Clause).

“who worked as a fireman for many years”

This phrase cannot stand on its own, but it adds detail and description to something – in this
case, it would provide background information about a person mentioned elsewhere within the
sentence. Notice that these commonly start with Relative Pronouns, like “who” or “where.”

These modifying details need to be attached to something or someone – they can’t just stand
on their own. Modifiers will never be independent; they are always attached to something else
from the main sentence.

Continuing with the fireman example, the modifying phrase might be attached to a sentence in
the following way:

“Jeremy, who worked as a fireman for many years, was able to rescue the cat
from the tall tree.”

Notice that the modifying phrase (or Parenthetical Clause) is not essential to the main sentence.
Take it out, and you’ll be left with “Jeremy was able to rescue the cat from the tall tree,” which
is still perfectly able to stand on its own.

Now, in the case above, the modifying phrase has not been “misplaced.” That’s because
“Jeremy,” the person who is being modified or described in more detail, is right next to the
modifying phrase “who worked as a fireman for many years.”

It’s really, really important that a modifier is right next to the person or thing it is modifying, so
that we know who the description is meant to “attach” to.

163
If this rule is broken, creating what we call a “Misplaced Modifier” error, then prepare for some
hilarious misinterpretations. Check out the ridiculous results of moving the modifying phrase to
a different location in the sentence, far away from “Jeremy”:

“Jeremy was able to rescue the cat, who worked as a fireman for many years,
from the tall tree.”

Heh heh. Silly sentence – cats can’t work as firemen! By placing the modifying phrase next to
“cat” instead of next to “Jeremy,” the modifier has been “misplaced,” and the meaning of the
sentence is now completely altered - not for the better!

The big trick to catching these errors every time they show up is to expect them. Any modifying
phrase should be considered “suspicious.” Treat these descriptive phrases as “guilty” until
proven innocent. Make sure the descriptive phrase is attaching to the correct noun.

Once you’ve found such a mistake, the next step is to look for answer choices that fix the
modifier by putting it next to the person or thing it was actually meant to describe. Use your
common sense and make sure that the modifying phrase matches up with (and is placed right
next to) something or someone that it’s actually meant to describe.

Let’s look at a Pretest question that might have sent you to this chapter:

While it panted and strained at its leash, the dog’s owner attempted to restrain the
puppy when it saw a squirrel at the dog park.

(A) [NO CHANGE]


(B) The dog’s owner attempted to restrain the puppy while it panted and strained at its
leash
(C) While it panted and strained at its leash, the puppy’s owner attempted to restrain it
(D) The dog’s owner, while it panted and strained at its leash, attempted to restrain the
puppy

First, notice the modifying phrase at the start of the sentence: “While it panted and strained at
its leash…”

This modifying phrase gives extra details about something, but what? The phrase can’t stand on
its own - until we read further, it’s unclear what or who this modifying phrase attaches to.

The way modifiers work is to attach to whatever person or thing they are closest to. In this
example, the nearest person or thing would be “the dog’s owner.” (Note the common trick of
using a possessive like “the dog’s owner” - many readers will only see “the dog” and not catch
the fact that “dog’s owner” actually refers to a person, not to an animal).

164
Isn’t it ridiculous to imagine a dog’s owner panting and straining at a leash? I certainly hope so…
otherwise, your life is a lot weirder than mine is. Yes, the original sentence contains a Misplaced
Modifier error!

Now that we’ve identified the mistake, start eliminating answer choices that repeat the same
flaw. Choice C makes the same Misplaced Modifier mistake by continuing to use the possessive
form to refer to a person instead of the puppy.

Choice D continues the Misplaced Modifier error, even though it shifts the modifying phrase
into the middle of the sentence. “The owner” is still the closest noun for the modifying phrase
to attach to.

Choice B is the correct solution: the phrase “while it panted and strained at its leash” is now
nestled up right next to “puppy,” the thing it describes. The Misplaced Modifier is fixed and the
meaning of the sentence is clear. Excellent!

Misplaced Modifiers Quick Reference


 Practice looking for Parenthetical Clauses, which are also called “modifying phrases”:
nonessential phrases that add extra detail or description. They cannot stand on their own.

 Modifying phrases / Parenthetical Clauses are often set apart from the rest of the sentence
by commas, and frequently begin with Relative Pronouns like “who” or “where,” or with “-
ing” words. Review Sentence Structure (Lesson 9) and Punctuation Marks (Lesson 8) for
more tips on Parenthetical Clauses and commas.

 The beginning or end of a sentence is usually the most common place to find a Misplaced
Modifier error; however, Misplaced Modifiers can also be hidden in the middle of the
sentence.

 Use common sense and logic to decide if a modifying phrase is describing the “wrong”
person or thing: Can cats be firemen? Do dog owners walk on leashes?

 Learn to appreciate the unintentional humor of Misplaced Modifiers so that you enjoy
looking for them and finding them.

 Fix these errors by moving modifying phrases so that they are right next to the person or
thing they are actually meant to describe.

165
Now let’s look at the second Pretest example of a Misplaced Modifier error:

Bursting with color, the pale hospital patient appreciated her friend’s gift of
flowers.

(A) [NO CHANGE]


(B) Bursting with color, the pale hospital patient’s friend’s gift of flowers
was appreciated.
(C) The pale hospital patient, bursting with color, appreciated her friend’s
gift of flowers.
(D) The pale hospital patient appreciated her friend’s gift of flowers, which
was bursting with color.

Notice that the sentence starts off with a modifying phrase again: “Bursting with color.” What,
exactly, is supposed to be “bursting with color?” Would a “pale hospital patient” be “bursting
with color,” or would a “gift of flowers” probably be more colorful?

In context, the “color” definitely should refer to the “gift of flowers.” That means the modifying
phrase “bursting with color” is in the wrong place! So, Choice A is out, as is Choice C (which has
a similar structure).

Choice B actually fixes the Misplaced Modifier error (it’s hard to tell with all the possessives, but
“bursting with color” actually does apply to the “flowers” in this sentence). However, compared
to the correct answer, it is very poorly-written; we would hope for a better option than the
multiple-possessive form “the hospital patient’s friend’s gift of flowers was appreciated,” which
is clumsy, confusing, wordy, and passive. See Lesson 13 on Redundancy & Wordiness for more
info.

Choice D is correct: It fixes the Misplaced Modifier mistake by placing the modifying phrase
“which was bursting with color” next to the thing it is meant to describe: the “gift of flowers.”
The meaning is clear, direct, and logical. Excellent!

166
Useful Links:
Redesigned SAT Practice Tests: http://www.cracksat.net/sat-tests/
SAT Online Practice Tests: http://www.cracksat.net/tests/
SAT Subjects Tests: http://www.cracksat.net/sat2/
SAT Downloads: http://www.cracksat.net/sat-downloads/
For more redesigned SAT information, please visit http://www.cracksat.net
SAT Grammar Practice Tests:

SAT Downloads:
Redesigned SAT Downloads: http://www.cracksat.net/sat-downloads/
SAT real tests download:
http://www.cracksat.net/sat-downloads/sat-real-tests.html
SAT official guide tests download:
http://www.cracksat.net/sat-downloads/sat-official-guide-tests.html
SAT online course tests download:
http://www.cracksat.net/sat-downloads/sat-online-course-tests.html
SAT subject tests download:
http://www.cracksat.net/sat-downloads/sat-subject-tests.html
PSAT real tests download:
http://www.cracksat.net/psat/download/

1000+ College Admission Essay Samples:


http://www.cracksat.net/college-admission/essays/
LESSON 12
Vocab Word Choice

Congratulations! You’ve made it through the majority of the “hardcore grammar rules”
contained in this book. This is perhaps the first lesson that’s not really about grammar in the
literal sense. This new topic - Vocab Word Choice - revolves around vocabulary: choices of
similar words with different tones or connotations.

Usually these questions look relatively short and sweet.- you’re just given four choices of
individual words. For example, from the Pretest:

The small house built in the field used the average building materials that
were available in the region: wood, stone, brick, and glass.

(A) [NO CHANGE]


(B) arbitrary
(C) stereotypical
(D) characteristic

Notice that all four words have something in common: they are a loosely-clustered group of
similar words that can overlap in meaning in certain contexts and usages. However, they all
have subtle differences in meaning and tone that can make a crucial difference when deciding
which word to use for the job.

Priorities of Vocab Word Choice Questions


The first priority in any Vocab Word Choice question should be to eliminate any words that
don’t have an accurate meaning that is fitting for the situation.

In the question above, we can safely eliminate Choice B, “arbitrary,” which means “based on a
random whim, rather than any reason or system.” The sentence tells us that the building
materials are selected specifically from what is available in the region, so “based on a random
whim” isn’t a good definition.

The second priority in a Vocab Word Choice question is to eliminate any words that don’t fit
the context.

On this ground, we can most likely eliminate Choice A, “average,” which fits best in a
mathematical context when dealing with sets of data, or in a more informal context when
comparing an individual to a larger group. This sentence does not present a particularly

178
mathematical or data-based context, so it’s unlikely that “average” will be the best word choice
for the job.

The third priority in a Vocab Word Choice question is to narrow your remaining choices down
by tone: the “positive,” “negative,” or “neutral” connotations, and the “formal” or “informal”
connotations of each remaining word choice.

Of our remaining choices, Choice C, “stereotypical,” has a negative connotation. This word
implies “an oversimplified idea of a particular type of person or thing.” Our sentence is neutral
and informational in tone, and there’s no negative connotation of being overly simple.

The best word for the job is Choice D, “characteristic.” It’s nothing fancy, but it’s perfect for the
job - a neutral, relatively formal word meaning “typical of a person, place, or thing.”

There’s one more priority to think about on a Vocab Word Choice question, which blends into
the next lesson on Wordiness and Redundancy. (These two topics are sometimes blended
together.)

For an example, what if Choice C had been “not unusual,” instead of the given option,
“stereotypical”? If that were the case, “not unusual” might pass the third priority’s test - it’s
quite neutral, without any negative connotations. However, the double negative of “not
unusual” would be unnecessarily wordy compared to the simpler option, “characteristic.”

In a Vocab Word Choice question in which two choices both fit the meaning, context, and tone,
then look for the most concise answer choice. It will usually be the correct one.

Levels & Difficulties of Vocab Words


These vocabulary questions aren’t generally focused on high-level “difficult” vocabulary. They
are mostly focused on the nuances of similar common words.

The majority of the vocab words will sound familiar to you - although you may find yourself
worrying about the exact usage of each word, when you’re faced with this type of question.

Remember, this topic isn’t so much about the “difficulty” of the vocab words as it is about their
exact meanings, appropriate contexts, connotation, tone, and brevity (which means “being
short, exact, and concise.”)

At the same time, you should be prepared to occasionally see more advanced vocabulary words.
Sometimes I think the test authors like to provide three “easy and familiar” words alongside a
single “challenging / advanced” vocab word - just to cause stress and chaos if you aren’t sure
whether the “advanced” vocab word would be a better choice than one of the three words that
you’re more familiar with.

179
Vocab Word Choice Quick Reference
 Recognize these questions by their simplicity and appearance. You’re given four similar -
but not identical - vocabulary words, and you’re supposed to select the best option for the
current context.

 First, focus on accurate meaning. Eliminate any words with unsuitable definitions.

 Next, eliminate any words that don’t fit the context of the sentence, based on the topic.

 Now focus on appropriate tone. Does the situation call for a word with positive, negative,
or neutral connotations? Is the setting more formal and academic, or more informal and
conversational?

 The final check should be for wordiness. All things being equal, the shorter answer is better.
This overlaps with Lesson 13 on Wordiness and Redundancy.

 To improve in this topic, read more books and articles each day to start learning the specific
contexts, connotations, and meanings of new words. Deliberately spend time developing
your vocabulary by looking up definitions and opening your mind to the subtle differences
in similar word choices. This will also help immensely with your Reading scores on the SAT
& ACT tests.

Following are a few details from my personal research into the SAT & ACT and their use of
Vocab Word Choice questions. One point became immediately clear: vocabulary is much more
important on the SAT Grammar test than it is on the ACT Grammar test.

180
ACT Vocab Word Choice
On the ACT Grammar section, this topic appears consistently, but in low frequencies. On
average, it is seen approximately just once per test (once per 75 questions) This means that
vocabulary questions are just over 1% of the whole ACT Grammar test, a very low percentage.

Here are a few word-choice selections from actual ACT Vocab Word Choice questions:

 illuminate, emanate, emulate, eliminate  arouse, stimulate, awaken, disturb

 voluminous, immeasurable,  heaps, a high number, hundreds, plenty


mountainous, large

SAT Vocab Word Choice


On the SAT Grammar section, Vocab Word Choice questions are much more common. On
average, it is seen approximately three times in a single test (three times per 44 questions).
That makes this topic about 7% of the whole SAT Grammar test - nearly six times higher in
importance than on the ACT.

Here a some selections of vocabulary words from actual SAT Vocab Word Choice questions:

- outdo, defeat, outperform, outweigh - taking on, undertaken in, overtaking, taking off
- satiated, fulfilled, complacent, sufficient from
- share advice, give some wisdom, proclaim our - blockade, scolding, interference, condemnation,
opinions, opine drag, reproof, deterrent, rebuke
- reducing, reductions, deducting, deducts - weirdnesses, deviations, oddities, abnormalities
- devour, dispatch, overindulge on, dispose of - keep pace, maintain the tempo, get in line, move
- austere, egregious, unmitigated, stark along
- sinister, surly, abysmal, icky - bolstering, amping up, arousing, revving up
- tight, firm, stiff, taut - elderly, old, mature, geriatric
- decreed, commissioned, forced, licensed - elaborated, developed, evolved, progressed
- confided, promulgated, imparted, unveiled - annihilating, eliminating, ousting, closing the door
- preserve, carry on, maintain, sustain on
- life-altering, galvanizing, intriguing, weird - potent, sturdy, influential, commanding
- site, cite, sight - scrupulous, super-rigorous, spot-on, intense
- emphatic, paramount, eminent, important - deformation, alteration, transformation,
- swear, subscribe, vow, promise modification
- persecuted, prosecuted
- pinnacle, triumph, culmination, apex

181
Let’s look at the other Pretest question that might have sent you to this chapter:

I prefer modern boat designs, and although my cousins enjoyed the interior
styling of the old steamship, I considered the decor fairly primordial.

(A) [NO CHANGE]


(B) elderly
(C) old-fashioned
(D) ancient

First, notice any words that don’t have an accurate meaning for the situation. Interestingly, all
of these words could possibly fit, since they all have a connection to “oldness.” However,
Choice A, “primordial,” has the dictionary meaning of “existing at the beginning of time.” It’s
unlikely that this is the ideal definition, since steamships did not exist at the beginning of time!

Next, move on to context. “Elderly” is a word most often seen in the context of people.
Although it is possible to use “elderly” to describe machinery or other objects, it’s an unusual
context, and chances are that we’ve got a more straightforward option available.

Finally, let’s move on to connotation. Although Choice D, “ancient,” could possibly be used in
this context, it has a connotation of “the very distant past.” Compared to our other option -
“old-fashioned” - it seems that “ancient” would imply a much older age, while “old-fashioned”
is more moderate option.

We could still try Choice D if we were uncertain, which would give us the phrase “fairly
ancient.” It should sound a bit ridiculous to you, because “ancient” implies extremely old, but
the word “fairly” is providing a more moderate tone that clashes with the more extreme
connotation of “ancient.”

That’s why Choice C is the correct solution: “Old-fashioned” has a straightforward accuracy
that fits the meaning, context and connotation of the “fairly old-fashioned” style of decoration
aboard an old steamship, without going to extremes.

Review and Encouragement


Don’t forget: the number-one way to improve your results on Vocab Word Choice questions is
to read more. Develop a curiosity for the meanings of words, their exact definitions, precise
usages and related synonyms. This will not only raise your SAT & ACT Grammar scores, it will
also make you a faster, better reader.

Not least of all, this will also make you a better writer and speaker, and you’ll find it easier to
express your own ideas with powerful, clear, persuasive, and precise language!

182
Useful Links:
Redesigned SAT Practice Tests: http://www.cracksat.net/sat-tests/
SAT Online Practice Tests: http://www.cracksat.net/tests/
SAT Subjects Tests: http://www.cracksat.net/sat2/
SAT Downloads: http://www.cracksat.net/sat-downloads/
For more redesigned SAT information, please visit http://www.cracksat.net
SAT Grammar Practice Tests:

SAT Downloads:
Redesigned SAT Downloads: http://www.cracksat.net/sat-downloads/
SAT real tests download:
http://www.cracksat.net/sat-downloads/sat-real-tests.html
SAT official guide tests download:
http://www.cracksat.net/sat-downloads/sat-official-guide-tests.html
SAT online course tests download:
http://www.cracksat.net/sat-downloads/sat-online-course-tests.html
SAT subject tests download:
http://www.cracksat.net/sat-downloads/sat-subject-tests.html
PSAT real tests download:
http://www.cracksat.net/psat/download/

1000+ College Admission Essay Samples:


http://www.cracksat.net/college-admission/essays/
LESSON 13
Redundancy & Wordiness

We’re on to another of my favorite topics: Redundancy & Wordiness! I like this topic because
students usually aren’t aware of its existence, but once they learn to recognize it, this can be a
fun and easy source of points on the SAT & ACT Grammar sections. Once you’ve learned to look
for these type of questions, they aren’t hard to find. You’ll wonder how you ever used to miss
them!

“Redundancy” means “the use of words that could be removed without loss of meaning;
unnecessary repetition.” That’s exactly what we’re looking for in this lesson: the unnecessary
repetition of information that could be removed without harming the clarity or meaning of the
sentence.

When a word is unnecessarily defined, repeated or rephrased into a nearly-identical synonym


within the same sentence, there is a strong chance that the question is focused on Wordiness
and Redundancy.

For example, if a sentence includes the word “contemporary,” it should not also include the
phrase “these days” in that sentence - because “contemporary” and “these days” both mean
exactly the same thing.

Redundant and BAD: “Contemporary movies produced these days often use an
excessive amount of computerized special effects. ”

Notice the redundancy of pairing “contemporary” with “these days” in the same sentence -
they have identical meanings.

For another example, the tests might try the following sentence on you:

Redundant and BAD: “The rare fish had blue (the color) and red stripes down its
tail fins. ”

Notice the inclusion of completely redundant information: “blue (the color)”. Of course blue is a
color; there’s no room for misinterpretation here. This is another example of a Redundancy
error, and the best solution is to completely remove the words “the color” from the sentence.
No loss of meaning or clarity will occur. (Believe it or not, there are actually Redundancy &
Wordiness choices like this on the SAT and ACT!)

192
Here’s a third example of a Redundancy error:

Redundant and BAD: “The company annually recycles a million tons of paper
each and every year.”

“Annually” already means “each year,” so keep “annually” and remove “each and every year.”

With Redundancy Eliminated: “The company recycles a million tons of paper


annually.”

“Redundancy” vs. “Wordiness”


So, is there a difference between “Redundancy” and “Wordiness”? Yes: the two concepts are
similar, but there are a few useful differences to be aware of.

As we’ve already seen, the concept of a Redundancy mistake is based on unnecessary repetition
that could be removed without harming the meaning or clarity of the sentence.

On the other hand, Wordiness is the mistake of using an excessive amount of words when a
simpler, more direct phrase is available.

Here is a rather basic example of Wordiness:

Wordy and BAD: “He walked along the path, and he was being accompanied by a
friend.”

To make the same point in about half the words, we could have said:

Less wordy and BETTER: “He walked along the path, accompanied by a friend.”

Notice we’ve removed the entire wordy phrase “and he was being,” without harming the clarity
or meaning.

The Word “Being” is a Common Giveaway of Wordiness


Any time the word “being” shows up in a SAT or ACT Grammar question, you should
immediately consider the possibility of a Wordiness error. There’s almost always a more
concise and direct way to rewrite the sentence without use of the word “being.”

As a rule of thumb, the SAT & ACT tests don’t like the word “being” - mostly because it tends to
create wordy, “passive” sentences. (We’ll examine the Passive Voice more in a moment).

193
Here’s an example of the dangers of the word “being,” taken from the Pretest:

The Vauxhall automobile, being designed to be economical, has recently


regained popularity among car collectors.

(A) [NO CHANGE]


(B) automobile being designed
(C) automobile, designed
(D) automobile, a car designed

Notice how when you remove the word “being,” the sentence’s meaning loses absolutely
nothing, and is just a little bit shorter and more direct? That eliminates Choices A and B. The
correct answer is Choice C. This is a “Wordiness” mistake.

Also, notice the Redundancy error of restating “a car” after “automobile” in Choice D - these
two words have the same definitions, and nothing meaningful is lost if we remove one of them
from the sentence.

Always remember this simple rule: the SAT and ACT Grammar tests dislike choices that use the
word “being” in them.

Hey, Shakespeare’s famous words were “To be or not to be,” not “Being to be or being not to
be,” right?

194
Passive Voice vs. Active Voice
“Passive Voice” vs “Active Voice” is a grammar topic that fits nicely into this lesson on
Wordiness.

What do “Active” and “Passive” mean in the context of grammar? Well, when someone does
something, that’s Active Voice. But, when someone has something done to them, that’s Passive
Voice. In English, we generally prefer the Active Voice - it’s typically more direct and concise -
and avoid the Passive Voice when possible.

This concept may be a unclear, so I’ve included some examples of Active vs. Passive Voice
below. Which versions seem better (clearer, more direct) to you?

Active: He drove the school bus.


Passive Voice: The school bus was being driven by him.

Passive Voice: Deer and rabbits being hunted with her bow and arrow was liked
by her. (LOL! So wordy!)
Active Voice: She liked to hunt deer and rabbits with her bow and arrow.

Active Voice: The subway traveled through many tunnels.


Passive Voice: Many tunnels were being traveled through by the subway.

Now let’s look at another example of Passive Voice and Wordiness from the Pretest:

Unethical political decisions are able to be protested by concerned


American citizens.

(A) [NO CHANGE]


(B) Unethical political decisions have the ability to be protested by
concerned American citizens.
(C) Concerned American citizens are able to protest unethical political
decisions made by politicians.
(D) Concerned American citizens are able to protest unethical political
decisions.

As written, this sentence is in the Passive Voice: “Decisions are able to be protested by…” Just
think about how poorly this idea is expressed: saying “decisions are able to…” makes it sound
like decisions have agency (the ability to decide and act independently). Instead, it should be
the citizens that have the agency and independent action.

195
Choice D fits our Active Voice improvement. Now the “citizens are able to protest,” and a
related (positive) side effect is that the sentence is shorter and less wordy. Also notice the
Redundancy in Choice C: it’s unnecessary to remind the reader that “political decisions” are
made by “politicians.”

So, let’s sum this lesson up and make a simple point of strategy very clear: on the SAT and ACT
Grammar tests, if picking between two choices that have no major grammar errors from the
previous lessons, you should always pick the shortest, most concise, most active and direct
answer that is offered to you. If there are two choices that seem free of errors, but one choice
is less wordy, then pick the less wordy option!

The shortest, clearest, and most direct answer wins – as long as there are no other grammar
errors in the sentence, of course. And, remember to always avoid the use of the word “being”
whenever possible.

Redundancy & Wordiness Quick Reference


 Start looking for Redundancy & Wordiness. It’s easier to notice when you’re already looking
for it.

 When choosing between two error-free answers, always favor the shortest, clearest, most
Active-Voiced answer that is available.

 “Redundancy” is the mistake of including repetitious information that can be removed


without harming the meaning or clarity of a sentence.

 It’s redundant to define a word twice in a sentence - for example, “annually each year,” or
“contemporary movies these days.”

 “Wordiness” is the use of long-winded phrasing that could be made simpler and more
direct.

 The word “being” is a common red flag for Passive Voice and Wordiness. Avoid use of the
word “being” whenever possible.

 Active Voice is when the noun does the verb. Passive Voice is when a noun has a verb done
to them. In English, the Active Voice is typically preferred over the Passive Voice.

196
Useful Links:
Redesigned SAT Practice Tests: http://www.cracksat.net/sat-tests/
SAT Online Practice Tests: http://www.cracksat.net/tests/
SAT Subjects Tests: http://www.cracksat.net/sat2/
SAT Downloads: http://www.cracksat.net/sat-downloads/
For more redesigned SAT information, please visit http://www.cracksat.net
SAT Grammar Practice Tests:

SAT Downloads:
Redesigned SAT Downloads: http://www.cracksat.net/sat-downloads/
SAT real tests download:
http://www.cracksat.net/sat-downloads/sat-real-tests.html
SAT official guide tests download:
http://www.cracksat.net/sat-downloads/sat-official-guide-tests.html
SAT online course tests download:
http://www.cracksat.net/sat-downloads/sat-online-course-tests.html
SAT subject tests download:
http://www.cracksat.net/sat-downloads/sat-subject-tests.html
PSAT real tests download:
http://www.cracksat.net/psat/download/

1000+ College Admission Essay Samples:


http://www.cracksat.net/college-admission/essays/
LESSON 14
Objective Questions

What is an Objective Question?


An Objective Question is my personal name for a very particular and interesting type of
question we will see on both the SAT & ACT tests. These types of questions are instantly
recognizable once you learn to identify them, yet each Objective Question also has the
potential to be completely unique.

Consider the majority of questions you face on the SAT & ACT Grammar sections, and you may
notice something interesting: most grammar questions do not actually give you any instructions.

Instead, most grammar questions simply present you with an underlined portion of a sentence -
with no instructions - then give you four answer choices to correct any possible grammar
mistakes in the underlined portion. Notice that no instructions are provided (and none are
necessary) because if you recognize and understand the appropriate rule(s) of grammar that
the question is testing, that will be enough information for you to arrive at a correct conclusion.

In contrast, Objective Questions require a set of instructions in order to be answered correctly.


These are questions that, without sufficiently clear directions, could not have a definitive “right
answer.”

For example, the following questions require specific instructions. Without these instructions,
our task would be unclear, and the single best answer choice would remain uncertain:

“Which choice best introduces the argument in the following sentence?”

“Which choice provides the most relevant and vivid sensory details?”

“Which choice most effectively combines the underlined sentences?”

To put this another way, Objective Questions assign you a specific writing task in their
instructions, and the correct answer choice is the one that best accomplishes those specific
instructions. Think about this, as you read a few more examples of possible Objective Questions:

“Which choice best summarizes the author’s central argument in the preceding
paragraph?”

“Which of the following placements for the underlined portion makes it most
clear that the business was continuing to expand its operations?”

206
“Suppose the writer’s goal had been to write a brief essay explaining the rise and
fall of an ancient culture in South America. Would this essay accomplish that
goal?”

Because they have specific instructions, you can see that each Objective Question has the
potential to be unique. In practice, many types of Objective Questions will reoccur in similar
forms across different tests. However, the possibility always exists to see a brand-new type of
Objective Question that you have never encountered before.

Furthermore, it takes a lot of testing experience to start encountering and recognizing


“repeated” Objective Questions. For these reasons (and more), I find it easier to simply treat
each and every Objective Question as its own unique experience, rather than trying to
generalize them all into a single inflexible approach.

Clearly, we must read the detailed instructions for each of our Objective Questions very
carefully, and avoid making any foolish assumptions about the nature of our task, or we will get
many of them wrong.

Why Do I Call Them “Objective Questions”?


Think about the name itself. Objective: a thing aimed at or sought, a goal. A mission, a specific
task to accomplish.

Doesn’t this perfectly describe the type of question we’ve just seen? We’re given a specific goal,
a mission, a set of instructions to follow.

I had to make up a name for this type of question, and I thought “Objective Questions”
captured the essence pretty nicely. Feel free to make up your own name for them, if you prefer,
and email it to me at Help@LovetheSAT.com!

Another Definition of “Objective”


There’s also a useful alternative meaning for the word “Objective”: not influenced by personal
feelings or opinions.

Contrast this against its antonym (opposite word), “Subjective,” meaning “based on personal
feelings, tastes, or opinions.”

This is another reason I like the name “Objective Questions,” because we must follow the
instructions we’re given, and must not be influenced by our own preferences, opinions, or
assumptions.

207
Objective Questions: A Research-Based Approach
I’ve conducted painstaking exclusive research into the details of Objective Questions to make
this lesson more informative, accurate, and useful. Below are some of the key takeaways I’ve
discovered. These secrets are not common knowledge, even among SAT and ACT prep tutors.

Amount of Objective Questions per Grammar Test


On the SAT Grammar test, there are an average of 9 Objective Questions per 44-question
Grammar test. This makes Objective Questions about 20% of all SAT Grammar questions - a
sizeable percentage of your final score. On some tests, this can drop as low as 15% or rise as
high as 30%.

It seems that the ACT has a slightly lower percentage of Objective Questions on average - about
13 out of 75 questions, or 17% of all questions, with a minimum of about 12% and a maximum
of about 20% per test. Still, they make a significant contribution to your final ACT Grammar
score. The ACT Objective Questions are also slightly less predictable and contain a higher
percentage of “unique” Objective Questions (see next page).

208
Most Common Types of Objective Questions by Test
According to my personal research, here are the most common types of Objective Questions by
test, with their approximate percentage of all Objective Questions listed after each type:

The SAT’s 8 Most Common Types of The ACT’s 6 Most Common Types of
Objective Questions: Objective Questions:

1. Most effectively combines the two sentences 1. Very specific, unique instructions (24%)
(20%) 2. Provides relevant / specific supporting
2. Supports main point of paragraph or gives information (22%)
relevant / supporting details or examples for 3. “Suppose the writer’s goal had been to
paragraph (18%) write about [X topic]. Would this essay
3. Sets up or supports the information, accomplish that goal? (19%)
examples, or argument that precedes or 4. Introducing or concluding the topic of a
follows (15%) passage, paragraph, or sentence (14%)
4. Accurately and effectively represents 5. Transitioning between sentences or
relevant info in a graph, table, or figure (12%) paragraphs (11%)
5. Very specific, unique instructions (12%) 6. If [specific sentence] were deleted, the
6. Provides best introduction or conclusion to a paragraph would primarily lose… (11%)
sentence, paragraph, or passage (10%)
7. Matching the existing style and/or tone (8%)
8. Transition between paragraphs, sentences,
or the ideas within a sentence (4%)

Objective Questions Can Overlap with Other Grammar Topics


Note that some Objective Questions overlap with the other Grammar rules presented in the
lessons of this book.

For example (and as you can see in the lists above), a common type of Objective Question will
focus on creating an effective transition; this can be between sentences, paragraphs, or even
two halves of the same sentence. These questions can essentially be seen as variations on
Conjunctions and Transitions questions (Lesson 7), but may also involve Punctuation Marks
(Lesson 8), Sentence Structure (Lesson 9), Parallelism (Lesson 10), Misplaced Modifiers (Lesson
11), Wordiness (Lesson 13) or any other Grammar topic we have covered in the preceding
lessons.

209
Preview of Lessons 15 & 16:
Common Subcategories of Objective Questions
For the moment, we’re most interested in the varieties of Objective Questions that have
“special instructions,” as in the lists of common question types shown above.

However, there are two common “subcategories” that we will study in the next two lessons of
this book. These subcategories are called “Adding or Removing Sentences” (Lesson 15) and
“Moving Sentences and Paragraphs” (Lesson 16). I have not included these subcategories
within the lists above, because the next two chapters will cover them in more detail.

These two common subcategories of Objective Questions occur frequently enough that I
eventually broke them out of this lesson and into their own special chapters with specific
techniques and strategies to deal with them.

Look for these two subcategories of Objective Questions in the following two lessons. For now,
we’ll continue to focus on techniques for the more “unique” varieties of Objective Questions
that we will find within the SAT & ACT tests.

Strategies for Answering Objective Questions


The most effective workflow for Objective Questions is flexible enough to adapt to multiple
different situations and question types, and looks something like this:

1. Recognize Objective Questions on sight.


2. Read them carefully.
3. Underline the most important keywords and instructions.
4. Eliminate the 2 worst answers first.
5. Double-check the instructions again.
6. Make your final choice based on the instructions.
7. Remain “objective.” Give what they ask for, not what you think would be best.

210
Try these strategies out on the following Pretest question.

“The Cloisters” is a museum in New York City that is constructed of four covered
walkways from Europe - named the Cuxa, Bonnefont, Trie, and Saint-Guilhem,
respectively - that were disassembled and transported to American soil between
1934 and 1939.

Which of the following choices provides the most relevant and useful supporting
details for the sentence?

(A) [NO CHANGE]


(B) a continent located in the Northern Hemisphere of the globe
(C) “cloisters” is another word for “covered walkways”
(D) moved from their places of origin

Let’s try following the steps I’ve given in the strategy above. First, we can easily recognize that
this is an Objective Question by the specific instructions that are provided beneath the
sentence. Let’s read them carefully. It seems that the most important instructions are “relevant
and useful supporting details.”

On that basis, let’s eliminate two of the worst choices. This would probably be Choice B
(because the main sentence is neither focused on the continent of Europe nor on giving
geographical details about its location) and Choice D (this information is Redundant (see Lesson
13) because the rest of the sentence already makes clear that the walkways were removed
from their places of origin).

Check the instructions one more time, to be safe. Right - we’re looking for “relevant and useful
supporting details.” Choice C is relevant but not useful, because it simply defines the word
“cloisters” as “covered walkways” - and the sentence has already provided that information, so
it’s also a case of more Redundancy.

Would Choice A, “No Change”, be acceptable? Yes - it provides the specific names of the four
walkways, which is new info (therefore “useful”) and relevant to the topic as well. Choice A is
the best answer.

211
Objective Questions Quick Reference
 Objective Questions are those that give specific directions within the question.

 The instructions for these questions can be unique, although many will fall into common
categories.

 Always underline the most important parts of the instructions before working on the
answer choices.

 Follow the instructions exactly. Never assume you already know what they want, or you will
occasionally misread and lose valuable points.

 Objective Questions can also include elements of other grammar topics within them. It’s up
to you to notice and recognize the other grammar rules from this book - as always, the tests
will not tell you.

 Use patient elimination to narrow down your answer choices first. Don’t rush to pick an
answer.

 Always double-check the instructions before giving your final answer.

212
Let’s look at another Pretest question:

James Scawen was a British politician in the mid-1700s who was once pressured
by a large mob to reveal how he had voted in an important election, and his father
inherited a large estate near London.

Which of the following choices provides the most relevant conclusion to the
preceding sentence?

(A) [NO CHANGE]


(B) but he declined to share this information, stating that he must keep his vote
secret in order to remain “an independent man.”
(C) and in his later years, he developed Carshalton Park with improvements like
canals, mills, and grottoes.
(D) and he lived from 1734 to 1801.

Again, let’s apply the steps of the Objective Question strategy. It’s clear this is an Objective
Question because it has a set of specific instructions provided along with the question. Reading
the instructions carefully, we should underline the most important directions: to provide the
“most relevant conclusion” to the preceding sentence.

With that in mind, we can eliminate anything that doesn’t seem like a “conclusion” to the
“politics” and “mob pressure” that the sentence highlights. I’d get rid of Choice A, which is
about Scawen’s father, not Scawen himself. Next I would drop Choice C, which changes topic
from politics to the development of a park many years later.

Now, for safety’s sake, double-check the instructions: “most relevant conclusion.” Although
discussing his year of death (“1801”) is a form of conclusion, it’s not nearly as relevant to the
political topic of the sentence as Choice B is, which concludes the story of the mob’s pressure to
reveal his vote with Scawen’s response to the mob. That’s why Choice B is the best answer.

213
Useful Links:
Redesigned SAT Practice Tests: http://www.cracksat.net/sat-tests/
SAT Online Practice Tests: http://www.cracksat.net/tests/
SAT Subjects Tests: http://www.cracksat.net/sat2/
SAT Downloads: http://www.cracksat.net/sat-downloads/
For more redesigned SAT information, please visit http://www.cracksat.net
SAT Grammar Practice Tests:

SAT Downloads:
Redesigned SAT Downloads: http://www.cracksat.net/sat-downloads/
SAT real tests download:
http://www.cracksat.net/sat-downloads/sat-real-tests.html
SAT official guide tests download:
http://www.cracksat.net/sat-downloads/sat-official-guide-tests.html
SAT online course tests download:
http://www.cracksat.net/sat-downloads/sat-online-course-tests.html
SAT subject tests download:
http://www.cracksat.net/sat-downloads/sat-subject-tests.html
PSAT real tests download:
http://www.cracksat.net/psat/download/

1000+ College Admission Essay Samples:


http://www.cracksat.net/college-admission/essays/
LESSON 15
Adding or Removing Information

What are Adding or Removing Information Questions?


“Adding or Removing Information” questions are a commonly-seen subcategory of Objective
Questions (see Lesson 14) that follow a particular and recognizable format. The general idea
behind these questions is that the writer is trying to decide if a particular sentence (or shorter
phrase) should be added or deleted from the passage. The answer choices will always present
two “Keep” options and two “Remove” options, and each of the four choices will provide
specific reasons and justifications to either keep or delete the sentence from the passage.

These questions are easy to identify, and with a little patience can be an easy source of points
without needing to master any of the more technical rules of Grammar. Adding or Removing
Information questions are intermediate in difficulty: the concept is easy to understand and
recognize, but the finer details of each problem take some time and patience, or you’ll miss key
details and get more questions wrong than you should.

How to Recognize Adding or Removing Information Questions


There are two ways to recognize Adding or Removing Information: The question format and the
answer choices format.

The questions follow a pattern something like this:

At this point, the writer is considering adding the following true statement:
“[Statement X]”. Should the writer make this addition here?

The answers follow a pattern something like this:

(A) Yes, because [Reason A].


(B) Yes, because [Reason B].
(C) No, because [Reason C].
(D) No, because [Reason D].

Note that these questions are not about “where” to put a sentence - that is a topic covered in
the next lesson, Moving Sentences & Paragraphs. Instead, Adding or Removing Information
questions follow a very specific pattern, and will never allow you to move a sentence, only

221
decide whether to include it or remove it. The answer choices always contain the “Yes, Yes, No,
No” or “Keep, Keep, Delete, Delete” pattern.

If this pattern is not met, it is not an “Adding or Removing Information” question, but rather
something else - most likely either a Moving Sentences & Paragraphs question or possibly
another type of Objective Question.

Examples of the Most Common Questions


Here are some examples of the most common types of Adding or Removing Information
questions on the SAT & ACT Tests.

SAT Questions: ACT Questions:

 The writer is considering deleting the  At this point, the writer is considering adding
underlined sentence. Should the sentence the following true statement: [new
be kept or deleted? statement]. Should the writer make this
 The writer is considering adding the addition here?
following sentence: [New sentence]. Should  The writer is considering deleting the
the writer make this addition here? preceding sentence. Should the sentence be
 The writer is considering adding the kept or deleted?
following information: [new information].  The writer is considering deleting the
Should the writer do this? underlined portion of the sentence. Should
 The writer is considering adding the the underlined portion be kept or deleted?
following graph: [graph/chart]. Should the
writer make this addition here?
 The writer is considering revising the
underlined portion of the sentence to read:
[Revised sentence]. Should the writer make
this change?

222
Minor Variations to the Question Style
As you may notice in the lists above, there are several minor variations to these questions that
you may encounter. One variation is to consider adding or deleting phrases within a sentence,
instead of a complete sentence:

At this point, the writer is considering deleting the underlined portion of the
sentence…

Or, sometimes you’ll be asked to consider adding a phrase to the beginning or end of a
sentence:

The writer is considering adding the following phrase to the end of the sentence…

Or, about starting or ending the essay with a new statement:

The writer is considering beginning the essay with the following true statement…

Finally, there is a rare variation about adding charts or graphs (in my experience, it is only the
SAT test that will ask about adding or removing a chart or graph):

The writer is considering adding the following [chart/graph] at this point in the
passage…

Despite these small changes to the wording or the question, all other elements are the same -
particularly the answer choices, which will continue to follow the characteristic pattern of:

(A) Yes, because [Reason A].


(B) Yes, because [Reason B].
(C) No, because [Reason C].
(D) No, because [Reason D].

In other words, despite some minor stylistic differences, the basic concept of all these question
variations is exactly the same as every other Adding or Removing Information question.

223
Examples of Common Answer Choices
Take a look at this list of common reasons for choosing “Keep” or “Delete” on Adding or
Removing Information questions. You don’t need to memorize this list - it’s just to give you a
clearer idea of the incredible variety of specific reasons to either “Keep” or “Delete”
information on the SAT & ACT tests. Always remember, the little details matter a lot to the right
answer!

- Delete, because the sentence interrupts the - Delete, because the sentence blurs the
flow of the paragraph. paragraph’s focus.
- Delete, because the sentence repeats - Keep, because the sentence elaborates on
unnecessary information. a specific topic in the paragraph.
- Delete, because the sentence is not related - Delete, because the sentence restates what
to the main topic of the passage. has already been said.
- Keep, because the sentence provides useful - Keep, because the sentence introduces an
examples. idea that becomes the focus for the rest of
- Delete, because the sentence introduces the essay.
irrelevant information to the paragraph. - Delete, because the sentence uses negative
- Keep, because the sentence provides terms such as lonely and forlorn to describe
relevant supporting info to the paragraph. the narrator’s experience.

- Keep, because the sentence sheds light on - Delete, because the sentence provides a
the main topic of the passage. level of detail that is inconsistent with the
level of detail in the rest of the essay.
- Delete, because the sentence does not
provide an accurate interpretation of the - Keep, because the sentence establishes a
data in the graph. correlation that is significant to the rest of
the essay.
- Keep, because the sentence provides
additional support for the main topic of the - Keep, because the sentence provides
paragraph. evidence to support a relevant claim.

-Keep, because the sentence serves as an - Delete, because the sentence contradicts a
effective counterargument to a claim central idea in the passage.
expressed in the passage. - Delete, because the sentence makes an
- Keep, because the sentence reinforces the informal observation that is not consistent
passage’s main point. with the essay’s tone.

- Delete, because the sentence undermines - Keep, because the sentence elaborates on
the passage’s claims. the preceding sentence’s point.

224
Similarities & Differences Between the SAT & ACT
In regards to “Adding or Removing Information” questions, there is little noticeable difference
between the SAT and the ACT tests. The styles of questions and answer choices are almost
indistinguishable between the two tests - even the most highly-trained tutor would have a hard
time telling if one of these questions came from the SAT or the ACT.

The percentage of these questions is also similar on the two tests: between about 4% and 8% of
the Grammar test. So, for all test-takers, these questions make a small, but noticeable,
contribution to your final score.

Essential Strategies For These Questions


The most important thing to remember when confronting these types of questions is that the
details of the answer choices matter a lot! Even a single wrong word in an answer choice is
enough to make the whole answer wrong.

Also, these questions are much more focused on content and context, and much less focused
on the rules of grammar - which makes them more similar to “Critical Reading” questions than
to “Grammar” questions. In fact, almost none of these questions have anything to do with
actual grammar rules. So,focus your attention on content, context, and meaning.

It’s very hard to “read the test’s mind” on these questions and jump straight to the correct
answer. So, I recommend against trying to find the right answer directly. Instead, strategic
Elimination is key. Don’t try to “pick” the right answer to these questions; rather, narrow down
your options by identifying weak or questionable points in each answer choice and crossing
them out. If any part of an answer choice is wrong, then the whole answer is wrong.

When searching for these weak points, use both broad strokes of “keep or delete” along with
the finer details buried within the answer choices. Often, one of these approaches will help
more than the other on a given question, but this will often change from question to question.
Remain flexible in the order you attack the answer choices, attend carefully to small details of
context, and proceed carefully by elimination.

If you’ve narrowed down as much as you can, but run out of ideas to finish the question off,
then take a final look and try to avoid any needless repetition or review of previous information.
Speaking broadly, both the SAT and ACT prefer to avoid repeating information that has already
been given, as we’ve seen in Lesson 13 on Redundancy & Wordiness.

225
Adding or Removing Information Quick Reference
 These types of questions can be easily recognized by their wording (“The writer is
considering adding the following information” and similar variations) and their answer
choices (“Keep / Keep / Delete / Delete” and similar variations).

 These questions have almost nothing to do with “grammar rules” and are usually not very
hard for careful and patient readers, but they do take patience and attention to the fine
details of the answer choices.

 Eliminate some answer choices before making your decision. Don’t leap to a final answer
before you eliminate most of your other options.

 Don’t decide your answer exclusively on the basis of “Keep/Delete” or “Add/Remove.” You
may be occasionally surprised what the SAT & ACT think about this. Be sure to investigate
the details of the answer choices and proceed by careful, steady elimination.

226
Let’s try a pair of Pretest questions based on the following paragraph:

[1] Of these, polyethylene is the most common type of plastic, accounting for 34% of the
world’s total plastics market. It has a startlingly-wide range of applications; for example,
ultra-high-density polyethylene is used in hip joint replacements, industrial shipping,
and ice-skating rinks. [2] On the other end of the spectrum, low-density polyethylene is
often used for plastic wraps, tubing, and food storage. In between these “high-density”
and “low-density” uses, polyethylene can also be found in film applications, water
plumbing, milk jugs, garbage containers, and children’s toys.

1. The writer is considering adding the following sentence as an introduction to the


paragraph.

Plastics are one of the most common and useful construction materials in modern global
industry, and are produced in a wide range of varieties.

Should the writer make this addition here?

(A) Yes, because the sentence provides an essential supporting example to the paragraph.
(B) Yes, because the sentence effectively introduces the main topic of the paragraph.
(C) No, because the sentence contradicts a central point made within the paragraph.
(D) No, because the following sentence (“Of these, polyethylene is…”) already provides a
more effective introduction to the paragraph.

2. At this point, the writer is considering deleting the underlined sentence. Should the
writer make this change here?

(A) Yes, because it provides an excessive list of examples for a term that has already been
defined.
(B) Yes, because it interrupts the overall flow of the paragraph.
(C) No, because it provides effective support for an essential counterargument within the
paragraph.
(D) No, because it provides examples that are used as a key point for a comparison
within the paragraph.

227
Here are explanations for the two questions.

Question 1 Explanation
Would this sentence provide an effective introduction to the paragraph? Perhaps. Let’s analyze
our options.

At a minimum, the new sentence is related to the topic of “polyethylene plastic” and doesn’t
contradict any central points within the paragraph. That eliminates Choice C. On the other hand,
it would be a stretch to call this new sentence a “supporting example,” so let’s eliminate Choice
A.

Choice B and Choice D offer two possible views on the same idea: either this sentence is an
effective introduction to the paragraph, or the following sentence is already a better
introduction. However, the original first sentence (“Of these, polyethylene is…”) makes an
unclear reference to whatever “these” are - a Pronoun-Antecedent Clarity error (see Lesson 4).

The new sentence would provide an effective introduction to the paragraph by providing the
antecedent “a wide range of varieties [of plastics]” leading into “of these, polyethylene is…”
Therefore, Choice B is the correct answer because the new sentence provides a clear pronoun
antecedent and an effective introduction to the next sentence and the paragraph as a whole.

Question 2 Explanation
In this question, we’re considering whether we should keep or delete the underlined sentence.
First, let’s take a close look at that sentence. It describes “the other end of the spectrum,” and
gives several specific examples of “low-density polyethylene.”

There is a good argument for keeping this sentence, because there is indeed a “spectrum” from
“high-density” to “low-density,” which this sentence helps clarify. Furthermore, the next
sentence focuses on examples between these “high-density” and “low-density” options, and
again refers to “low-density” - all of which would suddenly make less sense if we deleted the
underlined sentence.

Choices A and B are already looking pretty weak, because they propose sentence deletion. Let’s
take a closer final look to be sure: Choice A criticizes the “excessive list of examples” for a term
“that has already been defined.” The term in question must be “low-density polyethylene,”
which has not been defined elsewhere, and the list of examples is no more excessive than the
other lists of examples given in the paragraph for high-density polyethylene or the “in-
between” examples in the final sentence. Let’s eliminate Choice A.

Choice B claims the underlined sentence “interrupts the flow of the paragraph,” but it’s simply
not true: the sentence finishes the prior thought about the “startlingly-wide range” of “high-

228
density” and “low-density” polyethylene, before transitioning to the next sentence about the
mid-range of density. So, let’s eliminate Choice B.

Between the two “Keep” options, Choices C and D, we can see that Choice C refers to a
“counterargument” within the paragraph. But, there’s no counterargument - simply a spectrum
of options from high-density to low-density. On that basis, let’s eliminate Choice C and choose
the correct answer, Choice D, which hits the nail on the head: the underlined sentence
provides examples (“plastic wraps, tubing, and food storage”) that are used as a key point for a
comparison (the “spectrum” between “high-density” and “low-density” polyethylene).

Final Words
Remember, these questions can be easy if you treat them more like “Reading Comprehension”
questions. Details in the answer choices are key, so slow down and use an Elimination-based
strategy.

Don’t be afraid of these types of questions! They are easy to recognize, and, although they
often take a bit of extra time to answer, they can give you a nice break from the more
“technical” grammar questions.

229
Useful Links:
Redesigned SAT Practice Tests: http://www.cracksat.net/sat-tests/
SAT Online Practice Tests: http://www.cracksat.net/tests/
SAT Subjects Tests: http://www.cracksat.net/sat2/
SAT Downloads: http://www.cracksat.net/sat-downloads/
For more redesigned SAT information, please visit http://www.cracksat.net
SAT Grammar Practice Tests:

SAT Downloads:
Redesigned SAT Downloads: http://www.cracksat.net/sat-downloads/
SAT real tests download:
http://www.cracksat.net/sat-downloads/sat-real-tests.html
SAT official guide tests download:
http://www.cracksat.net/sat-downloads/sat-official-guide-tests.html
SAT online course tests download:
http://www.cracksat.net/sat-downloads/sat-online-course-tests.html
SAT subject tests download:
http://www.cracksat.net/sat-downloads/sat-subject-tests.html
PSAT real tests download:
http://www.cracksat.net/psat/download/

1000+ College Admission Essay Samples:


http://www.cracksat.net/college-admission/essays/
LESSON 16
Moving Sentences & Paragraphs

What are “Moving Sentences & Paragraphs” Questions?


Now we move on to the third and final subcategory of Objective Questions (introduced in
Lesson 14).

These new types of questions are called Moving Sentences & Paragraphs. They ask you to
consider moving a particular sentence (or paragraph) to a different spot in the passage.

Although the concept is easy to explain and identify, these remain some of my least-favorite
questions to do as a test-taker. Why? Even in the best of circumstances, they are time-
consuming and include a lot of information to consider - there is NO “quick and easy” way to
confidently and consistently answer these questions correctly.

In other words, compared to most other types of grammar questions, these take far longer to
answer correctly. Still, there’s no use burying our heads in the sand. Let’s take a careful look at
Moving Sentences and Paragraphs.

How to Recognize Moving Sentences & Paragraphs


“Moving Sentences and Paragraphs” have a very distinctive format for both the questions and
their answer choices.

Here’s some good news: the style of these questions for both the SAT and ACT tests is virtually
identical. If you can recognize them on one test, it’s quite simple to find them on the other test
as well.

Remember, these questions are all about moving an existing sentence or paragraph elsewhere.
Or, you may be adding a new sentence, but selecting where should it go in the passage.

The questions are quite distinctive - and we’ll take a look at them in a second - but their most
instantly-recognizable feature is their answer choices.

If you’re moving a single sentence within a paragraph, the answer choices look something like
this:

(A) where it is now.


(B) before sentence 1.
(C) after sentence 3.
(D) after sentence 6.

238
If you’re moving an entire paragraph to a new location within the passage, then the answer
choices will look something like this:

(A) where it is now.


(B) after paragraph 1.
(C) after paragraph 2.
(D) after paragraph 4.

You may also be given the opportunity to move a sentence to a new spot throughout the
passage, not just within a single paragraph, in which case the answer choice variations will look
something like this:

(A) Point A in paragraph 1.


(B) Point B in paragraph 2.
(C) Point C in paragraph 3.
(D) Point D in paragraph 4.

Variations and Styles of Question


Both the SAT and ACT tests share two types of “Moving Sentences & Paragraphs” questions:

Type 1: Move an existing sentence (or paragraph) to another location.


Type 2: Add a new sentence - but where will it be added?

Notice that Type 2 is noticeably different from Adding or Removing Sentences questions
(Lesson 15), because you are not given a choice of whether or not to include the new sentence.
Instead, the sentence will be added no matter what, and you just decide where it belongs. All of
these type of questions are about the placement of information in the paragraph or passage.

As I’ve mentioned earlier, these questions are virtually identical between the SAT & ACT tests.
In fact, I was hard-pressed to find any significant differences in my research.

Here are the exact phrasings the SAT and ACT tests use for these questions. You don’t need to
memorize these lists, but it’s good to be familiar with the question formats:

239
SAT Questions
SAT Type 1 (“Moving”) Questions SAT Type 2 (“Adding”) Questions
- To make this paragraph most logical, - The writer wants to add the following
Sentence X should be placed… sentence to the paragraph: [New Sentence.]
The best placement for this sentence is…

- For the sake of logic and cohesion of the


paragraph, Sentence X should be placed… - Where is the most logical place in this
paragraph to add the following sentence?
[New Sentence.]
- Think about the previous passage as a
whole as you answer question #. To make
the passage most logical, Paragraph X should - To improve the cohesion and flow of this
be placed… paragraph, the writer wants to add the
following sentence. [New Sentence.] The
sentence would most logically be placed…

ACT Questions
ACT Type 1 (“Moving”) Questions ACT Type 2 (“Adding”) Questions
- For the sake of logic and cohesion, the best - Question # asks about the preceding
placement for [Sentence / Paragraph X] passage as a whole. The writer wants to add
would be… the following sentence to the essay: [New
Sentence.] This sentence would most
logically be placed at…

- The writer wants to add the following


sentence to the paragraph: [New Sentence.]
The sentence would most logically be placed
at…

Frequency of Appearance
In terms of frequency, these questions show up as an average of 4% of all Grammar-section questions on both
the SAT and the ACT, or “1 per 25 questions.” So, they make a small, but important, contribution to your score.

The ACT has a minimum of 2 “Moving Sentences & Paragraphs” per 75 questions, or about 3% of all questions.
It has a maximum of 5 per 75 questions, almost 7%. As mentioned, the average is 4%.

The SAT has a minimum of 1 “Moving Sentences & Paragraphs” per 44 questions, or about 2%. There is a max of
3 questions, which is about 7%. Again, the average is 4%.

240
Essential Strategies for Moving Sentences & Paragraphs
The number-one strategy on these questions is patience. PATIENCE! If you think you’ll be able to quickly
identify the right answer 100% of the time, you are very wrong. Even the most experienced tutors require extra
time to analyze the possibilities, eliminate answer choices, and draw a confident conclusion. Never rush. You
don’t have some special skill or intuition that allows you to get these questions right faster than everyone else -
I promise. Because no one does.

I like to start by analyzing the sentence that is being moved. Often I break that sentence down into two halves:
“how it starts” and then “where it goes next.” Then I write a short summary of each half. In other words, I split
the sentence into two halves and create a simple written summary of each half. Use this opportunity to look for
clues (keywords, a timeline, transition words, etc) that may help explain where the sentence should (or should
not) go.

The next step is to skim and analyze the entire paragraph for good places to put the sentence, while eliminating
a few of the obviously wrong answers. In this step, I’m trying to predict where the sentence might go, while I
eliminate a few bad answer choices along the way.

The only thing I can do next is to try moving the sentence to each of the remaining possible locations. Make
sure that, wherever you put it, the end of the previous sentence leads into the moved sentence, and that the
start of the next sentence after it is a good followup to the moved sentence.

One of the most common mistakes I see students make is rushing and failing to give enough context - they don’t
consider the entire paragraph, only small portions of it. It’s not unusual to have to reread the entire paragraph,
from the first word to the last word, to fully grasp the context. I have had many good students express shock
when they realize “Oh, wow, I really needed to go back that far and re-read the whole darn thing?”

A Note on Time Management


Because of the necessary time commitment, it’s wise to leave Moving Sentences & Paragraphs questions for
last on your timed tests. They are a low percentage of points (about 4% of your score), but a high consumption
of your time (often taking 3x-5x longer than a typical grammar question). Circle back around to these questions
when you are finished with all the other questions in the section.

Let’s try out these strategies on the following Pretest question…

241
Think about the paragraph below as you answer the following question:

[1] The Japanese battleship Nagato was built in 1920 as the most powerful capital ship in the Imperial
Japanese Navy. [2] Her earliest noteworthy service was carrying supplies for the survivors of the Great
Kanto earthquake in 1923. [3] Nagato then participated in the Second Sino-Japanese War of 1937. [4]
Now, the wreck of the Nagato is considered one of the top ten shipwreck-diving spots in the world. [5]
She remained in service during World War 2 and was involved in the surprise attack on Pearl Harbor as
the flagship of Admiral Isoroku Yamamoto. [6] Impressively, Nagato was the only Japanese battleship
to survive the entirety of World War 2. [7] In 1946, the massive vessel was destroyed (with no sailors on
board) in the American nuclear weapons tests named “Operation Crossroads” held at Bikini Atoll.

1. To make this paragraph most logical, sentence 4 should be placed

(A) where it is now.


(B) before sentence 1.
(C) after sentence 5.
(D) after sentence 7.

Explanation: Luckily, this question gives us a relatively easy example of a “Moving Sentences & Paragraphs”
question. What makes it easy? Well, the clear sequence of years and events in a historical timeline makes our
job of re-positioning the sentence much easier. It’s logical to follow a timeline.

Let’s consider the sentence we’re moving, sentence 4:

“Now, the wreck of the Nagato is considered one of the top ten shipwreck diving spots in the
world.”

This sentence starts with a first half, “Now.” This is indicates that it comes after some other information from
the past, not before. It implies a timeline. It finishes with a second half that tells us the Nagato is now a
shipwreck. That gives us more context: we should look for a spot in the paragraph after the ship has been sunk
or destroyed.

Considering our options, Choice B makes very little sense. Putting this sentence at the very beginning of the
paragraph would ruin destroy the timeline by putting the sinking of the ship as the first event of the story.
Choice A isn’t great either: it interrupts two periods of operational service, during two separate wars, to discuss
the wreck of the ship. The sequence is illogical.

What about Choice C? Nope - it’s illogical - and goes against the timeline - to follow up a description of the
“wreck” of the Nagato with the sentence about “surviving” all of World War 2.

That leaves Choice D as the best answer. As the last sentence of the paragraph, it creates an effective timeline
and concluding sentence that “ends the life” of the ship. It’s also logical to follow up on the ship’s destruction in
sentence 7 with a description of its new status as a popular diving attraction.

242
Moving Sentences & Paragraphs Quick References
 These questions are easy to recognize. The questions are distinctive on their own, but the answer choices in
particular make this type of question very easy to spot at a glance.

 They are a low percentage of questions, but they are very time-consuming. Leave them for last, but don’t
forget about them.

 There’s no quick and dirty strategy that always works. It takes a lot of patience, analysis, and elimination,
even for pro tutors. You may have to test the sentence in each of the four possible locations.

 My favorite strategy is to break the sentence in half and write a quick summary of both the first half and
second half. This helps me make an educated decision about where the sentence belongs.

 Even experienced tutors don’t love these questions. Don’t expect perfection from yourself, and don’t get
frustrated. Just stay patient and practice as many of these as you can find.

Final Thoughts
These questions are usually very time-consuming, and they aren’t easy either. As a tutor, I see a lot of
frustration around these question types. Many students express the feeling that they are “taking too long” to
answer them. This is probably what causes so many students to rush through them, not considering and
analyzing all the available options, and the contents of the entire paragraph itself.

It’s unfortunately true that there is no super-secret strategy to ace 100% of these Moving Sentences &
Paragraphs questions. Patient analysis and practice is the only way. Stay calm and leave them for last. At least
these questions are easy to identify and avoid until the time is right to tackle them!

243
Useful Links:
Redesigned SAT Practice Tests: http://www.cracksat.net/sat-tests/
SAT Online Practice Tests: http://www.cracksat.net/tests/
SAT Subjects Tests: http://www.cracksat.net/sat2/
SAT Downloads: http://www.cracksat.net/sat-downloads/
For more redesigned SAT information, please visit http://www.cracksat.net
SAT Grammar Practice Tests:

SAT Downloads:
Redesigned SAT Downloads: http://www.cracksat.net/sat-downloads/
SAT real tests download:
http://www.cracksat.net/sat-downloads/sat-real-tests.html
SAT official guide tests download:
http://www.cracksat.net/sat-downloads/sat-official-guide-tests.html
SAT online course tests download:
http://www.cracksat.net/sat-downloads/sat-online-course-tests.html
SAT subject tests download:
http://www.cracksat.net/sat-downloads/sat-subject-tests.html
PSAT real tests download:
http://www.cracksat.net/psat/download/

1000+ College Admission Essay Samples:


http://www.cracksat.net/college-admission/essays/
Love the SAT Test Prep

LESSON 17
Idioms & Prepositions

Introduction
Here we are - the final lesson of this book. By this point you should understand and recognize a
vast amount of rules and common grammar situations. What you know from the first sixteen
chapters would be enough to earn you a stellar score on the SAT or ACT Grammar section -
quite possibly in the top 1% of test-takers around the world.

I highly recommend you master the first sixteen rules before you worry about this final chapter.
As you will soon see, the rules of Idioms & Prepositions are unpredictable, frustrating, and
impossible to fully prepare for.

Compared to this lesson, the other sixteen are far more logical - and more useful towards a high
SAT & ACT score. So be sure you’ve mastered the other sixteen rules first and left this lesson for
last.

What are “Idioms?”


Let’s start with a definition of the word “Idiom”:

An idiom is a group of words that have a specific meaning - which is established


by common usage - that can not be understood by analyzing the individual words
one at a time.

In other words, an idiom is “a phrase we say the way we say it, because that’s the way we say
it.” Pretty circular reasoning, but that’s what happens when a phrase develops its own unique
meaning through common usage over time.

For example, “it’s raining cats and dogs” is an example of a well-known idiom. Obviously it
doesn’t literally have anything to do with the animals “cats and dogs”; it’s just a phrase we’ve
established in English, through common usage, to mean “it’s raining very hard.”

Every language has its own idioms. These are often challenging to learn if it’s not your first
language. If a French exchange student asked you to explain exactly why “raining cats and dogs”
means “it’s raining very hard,” then you might realize that there’s no obvious explanation - it’s
just “the way we say it,” or “that’s just what it means.”

254
Love the SAT Test Prep

Even worse, if you make a change to the idiom - no matter how small - it usually won’t work
any more. Test for yourself: “it’s raining horses and goats”? Even a subtle change will ruin the
idiom: “It’s raining dogs and cats?” Try saying it out loud - it will sound “weird” to you because
the familiar, established order of the phrase has been reversed. This is another reason these
idioms are so difficult for most people to learn in a foreign language.

More Examples of Idioms


Here are a few more common idioms.

“Beat around the bush” = Avoid saying what you mean


“Get out of hand” = Get out of control
“Hit the sack” = Go to sleep
“Under the weather” = Sick

There are hundreds more. Your goal should not be to memorize them, but to start becoming
aware of them. Mastery of idioms comes through experience - typically, through reading and
conversation.

Prepositions: A Quick Review


Now, before we go further on Idioms, it may be helpful to head back to the Prelesson A on
Parts of Speech, but for now I’ll summarize the key information about that part of speech we
call prepositions.

Prepositions are “position” words. To name a few examples, these are words like:

- on - above - from
- under - below - behind
- around - in - among
- between - of - with

You may also recall the “Squirrel Test” for prepositions:

The squirrel runs up the tree.


The squirrel runs behind the tree.
The squirrel runs [preposition] the tree.

255
Love the SAT Test Prep

How do Idioms and Prepositions Relate?


So you may be thinking, “Great, I understood prepositions already, but what do those have to
do with Idioms?” The connection may not be clear.

However, many idioms are based on specific prepositions. For example, consider the idiom
“look it up in a dictionary.” Notice the preposition up, which forms an essential part of the
idiomatic phrase. Now imagine replacing up with down: “look it down in a dictionary.”

The idiom doesn’t work anymore, right? But why is that? I mean, there’s no logic to it: you
don’t look upwards at anything when you use a dictionary. If anything, you look more
downwards at the pages of the book. Still, you have to say “look it up” or the idiom doesn’t
sound right.

See, that’s why Idioms questions are so frustrating to teach. There’s almost never any logical
reason for them to be said the way they’re said… they just are. Using the wrong preposition
(like “down” instead of “up”) may completely invalidate the meaning of the idiom.

In other cases, using one preposition with the idiom may have one meaning, but using a
different preposition may create an entirely different meaning.

For example, if you say a house is on fire, that means it’s aflame. If you say a house is with fire,
it sounds wrong - we just don’t say it like that. The phrase “with fire” just isn’t a “thing.”

But wait - what if we change the preposition to “the house is under fire”? That has a completely
different meaning - the idiom “under fire” means that the house is being shot at.

All we’ve done is tried changing out a few different prepositions in the same idiomatic phrase,
but we’ve created dramatically different meanings. One preposition means one thing (“on fire”
= “aflame”), one means nothing (“with fire” = not how we say it), and a third means something
else entirely (“under fire” = “being shot at”).

I’m sure you can already start to see how these Idioms & Prepositions can get a little annoying
on a standardized test…

256
Love the SAT Test Prep

Recognizing Idioms & Prepositions Questions


It can be tough to know when you’re dealing with an Idioms & Prepositions question. One
possible approach is to master the first sixteen grammar rules, and then if the question doesn’t
fit into one of those sixteen rules, it may be an idiom.

Even better: if the answer choices are playing with prepositions (for example, swapping the
words for, into, around and on) then it is probably an Idioms & Prepositions question. But, not
all idioms are based on prepositions - although many are.

Sometimes, when I know the right answer to a grammar question, but can’t come up with a
good logical explanation for it - and this is very rare, probably less than 1 in 200 questions - I will
just throw my hands up and declare it an “idiom”: the answer is just the answer because “that’s
how we say it.”

257
Love the SAT Test Prep

Examples of Idioms & Prepositions Questions


Below, I’ve included a partial list of actual Idioms and Prepositions questions and answer
choices that I’ve found on real SAT and ACT tests. You do NOT need to memorize this list - I just
want you to browse through some examples. Also, I’ll warn you now: expect this list to seem
“random.” Idioms are a pretty bizarre topic with a huge variety.

I’ve italicized many of the prepositions in these idioms to call your attention to them.

- months of witnessing / months as if witnessing / months when witnessing / months


then witnessing
- in this regard / in which this regard / ones that / which
- chances of surviving / chances to survive
- famous for making / famous when he would make / famous as he made / famous
and made
- stripped away all in its path / stripped away all on its path
- one side of them / their one side
- far better / farther, better / far more of a better / a far, better
- had gone out of style / had went out of style / had went from style / had gone from
style
- cuts crossed / cut through
- would of / would have
- age-old / aging or older / old age / aging old
- serves to be / serves as / serves like / serves for
- read into / read about / read upon / read for
- practice by which / practice at which / practice from which / practice so that
- means through / means of / means from
- in order to / in order for
- go so far / go as far / go as far and / go so far as
- both of which / of which both / both of them / both
- subject to / subjected to / subjected from / subject for

258
Love the SAT Test Prep

Frequency of Appearance
Although these questions can be frustrating, there’s a silver lining: they are relatively
uncommon.

On the SAT, you will only see about 1 Idiom question per 44-question Grammar test. That’s less
than 3% of your entire score. Sometimes, an SAT test won’t have a single Idioms question on it.
At a maximum, I have seen 3 of them on a single SAT - but it’s rare to have so many.

On the ACT, these questions are only slightly more common. The average is about 4 Idioms
questions per 75-question Grammar test, or about 5% of your score. In my research I saw a
minimum of 3 Idioms questions per ACT, and a maximum of 5.

Since it’s impossible to fully prepare for these questions, it’s definitely good news that they are
one of the least-common question types on both the SAT & the ACT grammar tests.

Essential Strategies for Idioms & Prepositions


I’m going to be completely honest - there’s no way to be 100% prepared for these Idioms
questions. There are too many and their “rules” are just too random and illogical. So what can
you do to improve your chances?

First of all, read more! The more time you spend reading, the more familiar you’ll be with the
appearance and sound of common Idioms and Prepositions (plus this will help your SAT & ACT
Reading-section scores).

You can also be more active: search for lists of idioms on Google, print them off, hand-make
flashcards, or take written notes. Make your own lists of surprising idioms you’ve never seen
before and what they mean. You can even start working these new Idioms into your own
writing. Focus particularly on Idioms that depend on specific prepositions.

Review your Parts of Speech (Prelesson A) and know exactly how to recognize prepositions, so
you can immediately notice when the answer choices to a Grammar question are playing
around with prepositions.

In the bigger picture, remember to leave this entire topic for LAST. Only bother with Idioms
when you’re already 100% in control of every other rule in this book. In fact, you should be
scoring an average of 95%+ accuracy on your SAT or ACT Grammar tests before you even
consider worrying about Idioms. These are the single-most unpredictable questions on the
grammar tests. While every other topic can be completely prepared for and mastered, the
Idioms & Prepositions questions are vastly more unpredictable, and not subject to the normal,
logical rules of grammar.

259
Love the SAT Test Prep

Let’s try out one of our Pretest questions…

During his college interview, Jose was confused by several questions,


including one about his favorite types of animals, and another that asked,
“if you were a flavor of ice cream, what flavor would you be?”

(A) [NO CHANGE]


(B) confused at
(C) confused with
(D) confused for

Explanation: We can tell that this is probably a Prepositions and Idioms question by a quick
glance at the answer choices. Notice that they all change the preposition being used - from by
to at, with or for. This is exactly how most Idioms & Prepositions answer choices will appear.

It’s great that we’ve recognized the question type, but from here, it just comes down to our
knowledge, experience, and intuition. The answer is Choice A, No Change, because “confused
by” is just the way we say this phrase in English. You may say you’re “confused by the
question,” but never “confused at the question,” for example.

Hopefully, Choice A “sounds right” to you - because there’s no deeper explanation than that.
And that’s an Idioms question for you!

Idioms & Prepositions Quick Reference


 Master all other 16 Rules of Grammar first. Don’t even bother to study Idioms until you can
consistently hit 95%+ accuracy on any SAT or ACT Grammar section.

 Read more. Keep an eye out for Idioms in your reading.

 If you’re really ambitious and have the other rules locked down, search up a list of English
idioms on Google. Make handwritten notes or flashcards for idioms that are new to you.

 Start becoming aware of Idioms in your own writing, reading, and conversations. Try to use
more of them in your own essays and writing assignments.

 Review prepositions. Recognize this Part of Speech (Prelesson A) on sight. When the answer
choices play with different prepositions, this is a tip-off that the topic is probably Idioms.

 Don’t worry if you’re not getting these questions 100% correct. Focus on what is more
under your control: the other 16 Rules of Grammar.

260
Love the SAT Test Prep

Let’s try our luck on one more Pretest question…

Darla’s apartment was often difficult to fall asleep in, because it was
adjacent with a room of rowdy party animals who loved to stay up late and
blast their stereo system at maximum volume.

(A) [NO CHANGE]


(B) adjacent to
(C) adjacent at
(D) adjacent into

Explanation: Just like our previous question, we can tell it’s probably an Idioms & Prepositions
question because the four answer choices all play with different prepositions: with, to, at, and
into. From here’s just a matter of experience, intuition, and “what sounds right.” The correct
answer is Choice B, “adjacent to” - not for any specific reason, just because the preposition “to”
is what we pair with the word “adjacent” in English. And that’s just the way it is!

261
Useful Links:
Redesigned SAT Practice Tests: http://www.cracksat.net/sat-tests/
SAT Online Practice Tests: http://www.cracksat.net/tests/
SAT Subjects Tests: http://www.cracksat.net/sat2/
SAT Downloads: http://www.cracksat.net/sat-downloads/
For more redesigned SAT information, please visit http://www.cracksat.net
SAT Grammar Practice Tests:

SAT Downloads:
Redesigned SAT Downloads: http://www.cracksat.net/sat-downloads/
SAT real tests download:
http://www.cracksat.net/sat-downloads/sat-real-tests.html
SAT official guide tests download:
http://www.cracksat.net/sat-downloads/sat-official-guide-tests.html
SAT online course tests download:
http://www.cracksat.net/sat-downloads/sat-online-course-tests.html
SAT subject tests download:
http://www.cracksat.net/sat-downloads/sat-subject-tests.html
PSAT real tests download:
http://www.cracksat.net/psat/download/

1000+ College Admission Essay Samples:


http://www.cracksat.net/college-admission/essays/

You might also like